ChaseDream

标题: 请原谅我哪些钻牛角尖的题目。。T,T 感激。。 [打印本页]

作者: 199249712    时间: 2012-3-26 21:24
标题: 请原谅我哪些钻牛角尖的题目。。T,T 感激。。







生活不可能像你想象得那么好,但也不会像你想象得那么糟。我觉得人的脆弱和坚强都超乎自己的想象。有时,我可能脆弱得一句话就泪流满面,有时,也发现自己咬着牙走了很长的路。 ——莫泊桑
       单以这样的一句话作为开头,看高木直子的《一个人住第五年》的时候还在国内,那时觉得那样的生活根本不可能发生在我身上,连吃饭都要人陪着的我无法忍受一个人吃饭的感觉。所以后来,有很长的一段时间里我都没能适应一个人吃饭,一个人旅行,现在想想其实也没什么,这个世界运转速度那么快,没有人会在意你是不是一个人。以至于后来一个朋友问我是不是也得了社交恐惧症,我笑笑,其实不是,只是自己慢慢地变得懒了,懒得去经营一份感情,至于朋友,有那么几个就足够了,有些人天天在一起,也不见得是朋友。  
       好像这样久了,倒是会忘记开始遇到的困难,渐渐地变成自己生活的旁观者,看着生活平静地流淌。都说人是慢慢成长的,其实不是,人是瞬间长大的,就像是突然间沉淀一般,突然不会谈恋爱了或者说不想谈恋爱了,一个人生活单一却也不会觉得无聊,即便很多时候还是会迷茫却也不会觉得烦躁了。  
       去年的今天我在不一样的城市,背着不一样的书包,留着不一样的发型,走着不一样的路,想着不一样的事情,有着不一样的心思。谁说改变需要十年呢。  
       身边的牛人倒是不少,像是神抵一样的存在,我也只是羡慕想着反正自己也不会变成那样的人,直到有一天一个学长跟我聊起来,才知道原来他也有看不进去书的时候也有写论文写到想撞墙的时候,我们都忘了他们是用怎么样的一个代价才换取来了这样的一个人生。他说,如果你想要去实现梦想,孤独是你的必修课。如果不能沉下心来,就没有办法去实现它,因为那绝对不是一件容易的事情,孤独能让你更坚强,你必须找到自己的生活节奏。  
       有一个朋友喜欢每天喝一点酒,看一部电影然后准时睡觉;住在旁边的英国人神出鬼没有的时候早上才睡有的时候天刚黑就睡了;隔壁楼的一个男生每天天不亮就起来跑步,往往那个时候我才刚打算睡。  
       最近迷上一个人到处走,算不上旅行只是周围的城市走一遭,倒也不会花上太多时间准备,提起包就走了。我不会带上相机只是有兴致了拿出手机拍一拍,音乐倒是我走到哪里都不能丢的东西,只有音乐,能让看似漫长的等待变成曼妙的旅程,似乎自己跟整个世界都没有关系,只想当一片没有名字的云,徜徉在不知道名字的风景里。  
       我们都会找到自己的生活节奏,然后沉溺其中无法自拔。  
       很长一段时间里我都没有去书店,觉得那种“每个星期读一本书”对于我来讲是太遥远的东西。直到有一天我陪朋友去书店,他是一个买书就不会停的人,我也就跟着买了几本。回到家里看微博人人又觉得心里空拉拉的,索性就拿起书来看,也是在那一天我才发现,其实每个星期看一本书没那么难,那天我一下子把书看完,才觉得这样子的生活是充实的。  
       要么读书,要么旅行,身体和灵魂,必须有一个在路上。  
       我告诉自己现实容不得你拖延,拖延只会让我变得更焦虑而已,所以刚开始的时候我规定自己每天提早上床半小时,看上几十页书,很快就变成习惯了。有的时候我不得不感叹如果真的去做一件事情的话,那么这件事情没有那么难。当你真的想要做一件事情的时候,整个世界都会来协助你,就是这种感觉。  
       一个骑过川藏线的朋友说,只要出发,就能到达,你不出发,就哪里也去不了。如果你不能沉下心来,就什么也做不到。出发永远是最有意义的事,去做就是了。一本书买了不看只是几张纸,公开课下了不看也只是一堆数据,不去看就没有任何意义,反而徒增焦虑,行动力才是最关键的。  
       你也许也是这样,当你渴望找个人交谈的时候,你们却没有谈什么.于是发现有些事情是不能告诉别人的,有些事情是不必告诉别人的,有些事情是根本没有办法告诉别人的,而有些事情即使告诉了别人,你也会马上后悔。那么最好的办法就是静下来,真正能平静自己的只有自己。  
       没有人能免得了孤独,与其逃避它不如面对它。孤独并不是一件那么糟糕的事情,与嘈杂相比,一个人生活倒显得自得地多,倒也可以变成一种享受。或许至少需要那么一段时间,几年或几个月,一个人生活,不然怎么能找到自己的节奏知道自己想要什么。这是属于你自己的东西,是你的一部分,你听音乐时,坐地铁时,一个人走在马路上时,它就会流淌出来,让我觉得这个世界似乎在以另外一种形式存在着,我能够清晰地听到自己。  
       我们都生活在一个不那么如意的世界,当乌云密布我们就摇曳,但阳光总有一天会到来,等阳光照到你的时候,记得开出自己的花就行了,那个你与生俱来的梦想。有的时候梦想很远,有的时候梦想很近,但它总会实现的。我想一个人最好的样子就是平静一点,哪怕一个人生活,穿越一个又一个城市,走过一个又一条街道,仰望一片又一片天空,见证一次又一次别离。即便世界与我为敌,只要心还透明,就能折射希望。

转自心经贴某750NN的附件 很喜欢 就放上来了






黄色背景为正确答案 红色是我做错的地方。。 红色绿色标出的部分是偶在读题时候得分析。。

T,,T 泪流满面一下。。。






3. Because of the recent recession in Country A mostmagazines published there have experienced decreases in advertising revenue, so much so that the survivalof themost widely read magazines isin grave doubt. At the same time, however, more people in Country A are reading more magazines thanever before, and the numberof financiallysuccessful magazines in Country A is greater than ever.
Which one of the following, if true, most helps toresole the apparent discrepancy in the information above?
(C) Advertising is the main source of revenue onlyfor the most widely read magazines; other magazines rely on circulation for their revenue.
(D)Because of the recession, people in Country A have cut back on magazinesubscriptions and are reading borrowed magazines.

这道题我标出来了我认为是矛盾点的地方。。
当时读完题 偶是这么想的
矛盾点在于revenuenumber 然后我的定向就是number是假象
因为
1、如果我number上升 most widely readmagazinesnumber也会上升 那么矛盾就出现在了revenue number之间
2、而且感觉Cother不是很好 因为other不一定就涵盖了哪些financially successful magazines

T.T

Questions 5-6
Sales manager: Last year the total number of meals sold in our companyrestaurants was much higherthan it was the year before. Obviously consumers find our meals desirable.
Accountant: If you look at individual restaurants, however, you find that the number of meals soldactually decreasedsubstantially at every one of our restaurants that was in operation both lastyear and the year before. The desirability of our meals to consumers hasclearly decreased, given that this group of restaurants---the only ones forwhich we have sales figures that permit a comparison between last year and theyear before---demonstrates a trend toward fewer sales.

6. Which one ofthe following, if true, most seriously calls into question the accountant’s argument?
(A)The company’s restaurants last year dropped from their menus most of the newdishes that had been introduced the year before.
(E) Most ofthe company’s restaurants that were in operation throughout both last year andthe year before are located in areas where residents experienced a severeoveralldecline in income last year.
这道题再次果断选错了T.T  
E感觉是无关啊啊啊啊。。。跟收入有什么关系啊。。。
我的思路是 总体的多了平均的少了 说明除数→ 也就是饭馆的数量多了啊。。。
当时觉得A很明显T,T 结果错了。。


10. In order to pressure the government of Country Sto become less repressive, some legislators in Country R want to ban allexports from R to S. Companies in R that manufacture telecommunicationequipment such as telephones and fax machines have argued that exports of theirproducts should be exempted from the ban, on the ground that it is impossiblefor a country to remain repressive when telecommunication equipment is widelyavailable to the population of that country.
Which one of thefollowing is an assumption on which the argument given by the manufacturersdepends?
(B) Thetelecommunication equipment that would be imported into S if the exemption wereto be granted would not be available solely to top development officials in S.
(D) Of all exports that could be sent toCountry S, telecommunicationequipment would be the most effective in helpingcitizens of S oppose that countrys repressive government.
这道题其实是忧郁了一下 不知道排除哪一个。。。 就选了个有相似词语的、、、


14. If the government increases its funding for civilian scientificresearch, private patrons and industries will believe that such research hasbecome primarily the government’s responsibility. When they believe thatresearch is no longer primarily their responsibility, private patrons andindustries will decrease their contributions toward research. Therefore, in order to keepfrom depressing the overall level of funding for civilian scientific research,the government should not increase its own funding.
Which one of thefollowing is an assumption on which the argument relies?
(B) Any increase ingovernment funding would displace more private funding for civilian scientificresearch than it would provide.
(D) Civilian scientific research cannot beconducted efficiently with more than one source of funding.
也是犹豫了一下不知道选BD的哪一个。。感觉D是迷惑选项的一种形式。。。T.T 然后我中招了 是不是吖。。。。这道题我能感觉到D不对的地方,,但是说不出来。。


15. Dental researcher: Filling a cavity in a tooth is not a harmless procedure: itinevitably damages some of the healthy parts of the tooth. Cavities are harmfulonly if the decay reaches the nerves inside the tooth, and many cavities, ifleft untreated, never progress to that point. Therefore, dentists should notfill a cavity unless the nerves inside the tooth are in imminent danger fromthat cavity.
Which one of thefollowing principles, if valid, most strongly supports the researcher’sreasoning?
(C) Acondition that is only potentially harmful should not be treated using a methodthat is definitelyharmful.
(D)A condition that is typically progressive should not be treated using methodsthat provide only temporary relief.

啊啊啊 绿色标出来的部分T.T 我觉得C绝对不行啊。 一个是不完全无害 一个是绝对有害。。这怎么能对呢。。。T,T
作者: yiayia    时间: 2012-3-26 22:20
paradox题所给出的情况,都是真实情况,没有假象!题目就是要你解释两个“看似矛盾的现象"。
FACT1: the survival of the most widely read magazines isin grave doubt
FACT2:The number of financiallysuccessful magazines in Country A is greater than ever
NOTICE 这题为两个fact都找了理由
fact1的理由是:the recent recession in Country A mostmagazines published there have experienced decreases in advertising revenue。
fact2的理由是: more people in Country A are reading more magazines thanever before
突破口就在这两个理由上:
Advertising is the main source of revenue onlyfor the most widely read magazines; other magazines rely on circulation for their revenue.
author把报纸分为2类,一类是the most widely read magazines,另一类是others,其中others中的一部分成功了,且比例很大。
D选项说因为危机,人们取消了杂志订阅,开始读借的报纸。
这选项只提到了报纸订阅和解的报纸之间的关系,而没有提到两个the most widely read magazines 和others.属无关选项。
作者: yiayia    时间: 2012-3-26 22:31
再次分离推理的主干:
premise1: the number of meals soldactually decreased substantially at every one of our restaurants that was in operation both last year and the year before.
premise2:sales figures permit a comparison between last year and the year before
conclusion: The desirability of our meals to consumers hasclearly decreased.
答案直接削弱premise 2.sales figures  between last year and the year before 不可比,因为市场环境不一样,有没有觉得很像argument?
A说去年取消了前年的菜式,不是正好加强了Accountant的结论, meal吸引度下降了?
作者: yiayia    时间: 2012-3-26 22:38
第10题 D不是必要性假设,理解必要性假设,可以选用摘自sdacar的这一个比喻:
For the difference between sufficient assumption and necessargy assumption, here is an analogy:

Suppose you have city A and city B, separated by a river H.  If there are multiple bridges over the river H, each bridge is sufficient but not necessary to connect city A with city B. What is necessary is that at least one bridge over river H is working.

If there is only one bridge over the river H, then that bridge is both sufficient and necessary to connect city A with city B.

If there there are two rivers C and D separate the cities A and B, and there is one bridge over C and many bridges over D, then the bridge over C is necessary (but not sufficient) to connect city A and B, while all the individual bridge over D is neither necessary nor sufficient to connect city A with city B. What is necessary is that at least one bridge over D is working!
即使 negative D,  Of all exports that could be sent toCountry S, telecommunication equipment would not be the most effective in helpingcitizens of S oppose that country’s repressive government. 也不能否定结论。
What if it is the second or third effective?

14题,上邪自己尝试negative一下,如果negative 选项后,结论不成立,那么这就是必要性假设。
作者: yiayia    时间: 2012-3-26 22:52
Background:Filling a cavity in a tooth is not a harmless procedure: it inevitably damages some of the healthy parts of the tooth.
Premise:Cavities are harmful only if the decay reaches the nerves inside the tooth, and many cavities, if left untreated, never progress to that point.
Conclusion:Therefore, dentists should not fill a cavity unless the nerves inside the tooth are in imminent danger from that cavity.
background 告诉我们 filling a cavity是有害的, 然后说cavities 很多不会危害健康,所以 牙医不应该fill a cavity 知道 cavity危害神经。
cavities 很多不会危害健康 是potentially harmful。
Thus, a condition that is only potentially harmful should not be treated using a methodthat is definitelyharmful.呵呵,其中有几题yiayia也错过,一定要好好分析错误的原因。 像牙医的最后一题,很有可能是没有读懂题意就选了。anyway, CR=reading+reasoning.
作者: babybearmm    时间: 2012-3-27 04:00
这题我也选的E,不过我跟yiayia的理解有点不一样....欢迎讨论

Sales manager: "total number of meals sold in our company restaurants"     N(total) ↑
Accountant: "the number of meals sold at individual restaurants with available data"        Ni ↓  (for i=1, 2, ...k, in which data is available)

Accountant's premise is above     Ni ↓  (for i=1, 2, ...k, in which data is available)
Accountant's conclusion is: "The desirability of our meals to consumers has clearly decreased"

Choice E provides an alternative cause:
NOT   desirability to consumers
BUT    (population income)i ↓        (for i=1, 2, ...k, in which Ni data is available)

Because people have less income, they have less purchasing power. So the decline in population income is an alternative cause.  Also, this choice carefully notes that "(restaurants) that were in operation throughout both last year and the year before", which clearly specifies the objects for the consideration - for i=1, 2, ...k, in which Ni data is available.

再次分离推理的主干:
premise1: the number of meals soldactually decreased substantially at every one of our restaurants that was in operation both last year and the year before.
premise2:sales figures permit a comparison between last year and the year before
conclusion: The desirability of our meals to consumers hasclearly decreased.
答案直接削弱premise 2.sales figures  between last year and the year before 不可比,因为市场环境不一样,有没有觉得很像argument?
A说去年取消了前年的菜式,不是正好加强了Accountant的结论, meal吸引度下降了?
-- by 会员 yiayia (2012/3/26 22:31:05)





作者: babybearmm    时间: 2012-3-27 04:15
太精彩了!

第10题 D不是必要性假设,理解必要性假设,可以选用摘自sdacar的这一个比喻:
For the difference between sufficient assumption and necessargy assumption, here is an analogy:

Suppose you have city A and city B, separated by a river H.  If there are multiple bridges over the river H, each bridge is sufficient but not necessary to connect city A with city B. What is necessary is that at least one bridge over river H is working.

If there is only one bridge over the river H, then that bridge is both sufficient and necessary to connect city A with city B.

If there there are two rivers C and D separate the cities A and B, and there is one bridge over C and many bridges over D, then the bridge over C is necessary (but not sufficient) to connect city A and B, while all the individual bridge over D is neither necessary nor sufficient to connect city A with city B. What is necessary is that at least one bridge over D is working!
即使 negative D,  Of all exports that could be sent toCountry S, telecommunication equipment would not be the most effective in helpingcitizens of S oppose that country’s repressive government. 也不能否定结论。
What if it is the second or third effective?

14题,上邪自己尝试negative一下,如果negative 选项后,结论不成立,那么这就是必要性假设。
-- by 会员 yiayia (2012/3/26 22:38:55)


作者: babybearmm    时间: 2012-3-27 04:31
14. If the government increases its funding for civilian scientific research, private patrons and industries will believe that such research has become primarily the government’s responsibility. When they believe that research is no longer primarily their responsibility, private patrons and industries will decrease their contributions toward research. Therefore, in order to keep from depressing the overall level of funding for civilian scientific research, the government should not increase its own funding.

Which one of the following is an assumption on which the argument relies?


(A) Governments should bear the majority of the financial burden of funding for civilian scientific research.

(B) Any increase in government funding would displace more private funding for civilian scientific research than it would provide.
(C) Private donations toward research are no longer welcomed by researchers whose work receives government funding.
(D) Civilian scientific research cannot be conducted efficiently with more than one source of funding.
(E) funding for civilian scientific research is currently at the highest possible level.


I go with B. Is B the correct answer?
My diagram below

(F1 = funding from the government, F2 = funding from private patrons and industries)

Premise:  if F1
↑,  then F2  
Conclusion:  in order to let F(total) not
,  should not make F1

Since   F(total) = F1 + F2
The conclusion requires: the decrease in F2 outweighs the increase in F1
Hence Choice B.

D is out-of-scope. We are interested in "
keep from depressing the overall level of funding", which is F(total). We do not care whether the research can be conducted efficiently.  
 
作者: babybearmm    时间: 2012-3-27 04:50
对了,这第10题我当时花了3'32'',当时完全没思路怎么prephrase,第一遍曾把所有选项都排除,然后推倒重来的。现在想想,我能不能这么理解?

In order to pressure the government of Country S tobecome less repressive, some legislators in Country R want to ban all exportsfrom R to S. Companies in R that manufacture telecommunication equipment
such as telephones and fax machines have argued that exports of their productsshould be exempted from the ban, on the ground that it is impossible for acountry to remain repressive when telecommunication
equipment is widely available to the population of that country.


Highlighted portion is the premise. premise是个条件命题,条件是"when telecommunication
equipment is widely available to the population of that country
",那assumption,就得是说"when"这个前提条件成立。

(B) The telecommunication equipment that would be imported into S if the
exemption were to be granted would not be available solely to top government officials in S.



太精彩了!

第10题 D不是必要性假设,理解必要性假设,可以选用摘自sdacar的这一个比喻:
For the difference between sufficient assumption and necessargy assumption, here is an analogy:

Suppose you have city A and city B, separated by a river H.  If there are multiple bridges over the river H, each bridge is sufficient but not necessary to connect city A with city B. What is necessary is that at least one bridge over river H is working.

If there is only one bridge over the river H, then that bridge is both sufficient and necessary to connect city A with city B.

If there there are two rivers C and D separate the cities A and B, and there is one bridge over C and many bridges over D, then the bridge over C is necessary (but not sufficient) to connect city A and B, while all the individual bridge over D is neither necessary nor sufficient to connect city A with city B. What is necessary is that at least one bridge over D is working!
即使 negative D,  Of all exports that could be sent toCountry S, telecommunication equipment would not be the most effective in helpingcitizens of S oppose that country’s repressive government. 也不能否定结论。
What if it is the second or third effective?

14题,上邪自己尝试negative一下,如果negative 选项后,结论不成立,那么这就是必要性假设。
-- by 会员 yiayia (2012/3/26 22:38:55)




-- by 会员 babybearmm (2012/3/27 4:15:02)




作者: babybearmm    时间: 2012-3-27 05:03
Background:Filling a cavity in a tooth is not a harmless procedure: it inevitably damages some of the healthy parts of the tooth.
Premise:Cavities are harmful only if the decay reaches the nerves inside the tooth, and many cavities, if left untreated, never progress to that point.
Conclusion:Therefore, dentists should not fill a cavity unless the nerves inside the tooth are in imminent danger from that cavity.
background 告诉我们 filling a cavity是有害的, 然后说cavities 很多不会危害健康,所以 牙医不应该fill a cavity 知道 cavity危害神经。
cavities 很多不会危害健康 是potentially harmful。
Thus, a condition that is only potentially harmful should not be treated using a methodthat is definitelyharmful.呵呵,其中有几题yiayia也错过,一定要好好分析错误的原因。 像牙医的最后一题,很有可能是没有读懂题意就选了。anyway, CR=reading+reasoning.
-- by 会员 yiayia (2012/3/26 22:52:31)



我同意yiayia的分析,觉得这题就是yiayia总结里特别强调的也是bible说的要小心形容词和副词。这题题干和正确选项能完美对应上的:
not harmless, inevitably damages   = definitely harmful
harmful only if ... = potentially harmful
作者: 199249712    时间: 2012-3-27 09:52
啊啊啊 我超喜欢这些思路!!~~

太感谢姐姐还有yiayia哥了~~!
偶先去上专业课晚点就把自己的改错总结放上来 mua~~
作者: teddybearj4    时间: 2012-3-27 12:08
好帖哇~顶之慢慢研究一下~
作者: 199249712    时间: 2012-3-27 13:12
3. Because of the recent recessionin Country A mostmagazines published there have experienced decreases in advertising revenue, so much so that thesurvivalof themost widely read magazinesisin grave doubt. At the same time, however, more people in Country A are reading more magazines thanever before, and the numberof financiallysuccessfulmagazines in Country A is greaterthan ever.
Which one of the following, if true, most helps toresole the apparentdiscrepancy in the information above?
(C) Advertising is the main source of revenueonlyfor the most widely read magazines;other magazines rely on circulation fortheir revenue.
(D)Because of the recession, people in Country A havecut back on magazinesubscriptions and are reading borrowed magazines.



分析:错的原因果然是找错了paradox。。我一直认为是同一种 mostwidely readmagazines
杂志的revenue&number的矛盾 昨天一直拗不过来这个弯 愣是想不通为什么是两个杂志比。。。  yiayia哥的分析好棒!
  【突破口就在这两个理由上:
Advertising is the main source of revenue onlyfor the most widely readmagazines; other magazines rely on circulation for their revenue.


Questions 5-6
Sales manager: Last year the total numberof meals sold in our companyrestaurants was much higherthan it was the year before. Obviously consumers find ourmeals desirable.
Accountant: If you look at individualrestaurants, however, you find that the number ofmeals soldactually decreasedsubstantiallyat every one of our restaurants that was in operation both lastyear and theyear before. The desirability of our meals to consumers hasclearly decreased,given that this group of restaurants---the only ones forwhich we have salesfigures that permit a comparison between last year and theyearbefore---demonstrates a trend toward fewer sales.

6. Which one ofthe following, if true, most seriously calls into question the accountant’s argument?
(A)The company’s restaurants last year dropped fromtheir menus most of the newdishes that had been introduced the year before.
(E) Most ofthe company’s restaurants that werein operation throughout both last year andthe year before are located in areaswhere residents experienced a severeoveralldeclinein income last year.


答案直接削弱premise 2.sales figures between last year and the year before 不可比,因为市场环境不一样,有没有觉得很像argument?
啊啊啊这个解释字字珠玑。。
A说去年取消了前年的菜式,不是正好加强了Accountant的结论, meal吸引度下降了?





10. In order to pressure the government of Country Sto become lessrepressive, some legislators in Country R want to ban allexports from R to S.Companies in R that manufacture telecommunicationequipment such as telephonesand fax machines have argued that exports of theirproducts should be exemptedfrom the ban, on the ground that it is impossiblefor a country to remainrepressive whentelecommunication equipment is widelyavailable to the population of thatcountry.
Which one of thefollowing is an assumption on which the argument given by themanufacturersdepends?
(B) Thetelecommunication equipment that wouldbe imported into S if the exemption wereto be granted would not be availablesolely to top development officials in S.
(D) Of all exports that could be sent toCountry S, telecommunication equipment would be the most effective in helpingcitizens of S oppose that country’s repressive government.




14. If the government increasesits funding for civilian scientificresearch, private patrons and industrieswill believe that such research hasbecome primarily the government’sresponsibility. When they believe thatresearch is no longer primarily theirresponsibility, private patrons andindustries will decrease their contributionstoward research. Therefore, in order tokeepfrom depressing the overall level of funding for civilian scientific research,thegovernment should not increase its own funding.
Which one of thefollowing is an assumption on which the argument relies?
(B) Any increase ingovernment funding would displace moreprivate funding for civilian scientificresearch than it would provide.
(D) Civilian scientific research can not be conducted efficiently with morethan one source of funding.
我想1410题应该是同一个原因。有没有效果跟她否定后削弱是不能同等的。。因此我在取非后不能削弱T.T



15. Dental researcher: Filling a cavity in a tooth is not a harmless procedure: itinevitably damagessome of the healthy parts of the tooth. Cavities are harmfulonly if the decayreaches the nerves inside the tooth, and many cavities, ifleft untreated, neverprogress to that point. Therefore, dentists should notfill a cavity unless thenerves inside the tooth are in imminent danger fromthat cavity.
Which one of thefollowing principles, if valid, most strongly supports theresearcher’sreasoning?
(C) Acondition that is only potentially harmfulshould not be treated using a methodthat is definitelyharmful.
(D)A condition that is typically progressive should notbe treated using methodsthat provide only temporary relief.

这道彻底懂了!!! 词的表达真的很重要啊
作者: 199249712    时间: 2012-3-27 13:12
再次感谢Yiayia哥 baby姐姐写出你们的思路给偶…………
作者: yiayia    时间: 2012-3-27 15:45
对了,这第10题我当时花了3'32'',当时完全没思路怎么prephrase,第一遍曾把所有选项都排除,然后推倒重来的。现在想想,我能不能这么理解?

In order to pressure the government of Country S tobecome less repressive, some legislators in Country R want to ban all exportsfrom R to S. Companies in R that manufacture telecommunication equipment
such as telephones and fax machines have argued that exports of their productsshould be exempted from the ban, on the ground that it is impossible for acountry to remain repressive when telecommunication
equipment is widely available to the population of that country.

Highlighted portion is the premise. premise是个条件命题,条件是"when telecommunication
equipment is widely available to the population of that country",那assumption,就得是说"when"这个前提条件成立。

(B) The telecommunication equipment that would be imported into S if the
exemption were to be granted would not be available solely to top government officials in S.



太精彩了!

第10题 D不是必要性假设,理解必要性假设,可以选用摘自sdacar的这一个比喻:
For the difference between sufficient assumption and necessargy assumption, here is an analogy:

Suppose you have city A and city B, separated by a river H.  If there are multiple bridges over the river H, each bridge is sufficient but not necessary to connect city A with city B. What is necessary is that at least one bridge over river H is working.

If there is only one bridge over the river H, then that bridge is both sufficient and necessary to connect city A with city B.

If there there are two rivers C and D separate the cities A and B, and there is one bridge over C and many bridges over D, then the bridge over C is necessary (but not sufficient) to connect city A and B, while all the individual bridge over D is neither necessary nor sufficient to connect city A with city B. What is necessary is that at least one bridge over D is working!
即使 negative D,  Of all exports that could be sent toCountry S, telecommunication equipment would not be the most effective in helpingcitizens of S oppose that country’s repressive government. 也不能否定结论。
What if it is the second or third effective?

14题,上邪自己尝试negative一下,如果negative 选项后,结论不成立,那么这就是必要性假设。
-- by 会员 yiayia (2012/3/26 22:38:55)





-- by 会员 babybearmm (2012/3/27 4:15:02)




-- by 会员 babybearmm (2012/3/27 4:50:03)

baby姐说的对,但全部排除掉的原因我认为是有些选项藏的太深,从反面看比较容易
必要性假设类negative就像比喻中的那样,把通过所必须的C桥给关闭了,那么肯定通不过了,也就是说结论不成立!
但如何negative是个技巧,需要练习。像B选项就是The telecommunication equipment that would be imported into S if the
exemption were to be granted would  be available solely to top government officials in S.
如果如上所述,on the ground when后面的条件不成立,结论也不成立。
正向思维有时候不如反向思维清晰。
作者: 199249712    时间: 2012-3-27 17:27
21. Historian:
background:We can learn about the medical history of individuals throughchemical analysis of their hair. It is likely, for example, that Isaac Newton’spsychological problems were due to mercury poisoning; traces of mercury werefound in his hair.
Analysis is now being done on a lock of Beethoven’shair. Although no convincing argument has shown that Beethoven ever had avenereal disease, some people hypothesize that venereal disease caused hisdeafness. Since mercury was commonly ingested in Beethoven’s time totreat venereal disease, if researchers find a trace of mercury in his hair, wecan conclude thatthis hypothesis is correct.

Which one of thefollowing is an assumption on which the historian’s argument depends?

(A)None of the mercury introduced into the body can be eliminated.
(B) Some people in Beethoven’s time did not ingestmercury.

这道题偶是这么分析的:
Mercury治愈venereal disease
Mercury存在于头发里
贝多芬头发含有Mercury 贝多芬得了venereal disease

我读完后的定向有两个:
1、 Mercury不会随着时间消失  消失了就检测不到这个病了也就说明不了什么问题了
2、 别的病不拿Mercury来治

后来对了答案后 理解的结果是:
Mercury不是一种普遍现象 如果吃Mercury是普遍现象那么也就不能说明venereal diseaseMercury的特殊性
但是感觉B的表述不是很清楚的说……因为some毕竟含有特例的因素在里面
那我也可以因此理解为:some特殊群体没事就吃Mercury。。跟我人民大众治病良方无关0 0
yiayia哥帮我解释一下这里词的用法好不好。。我感觉我对词的精准性一直理解地不是很到位 包括上一组的那个牙医的题 你一讲我突然就明白了那个inevitably & definitely 之前打死也想不通。。偶最近也在看bible。。不过还在理解中T.T 一直内化不了成自己的知识体系 求传授……5555……】

最后分析了一下自己错误的原因。。。感觉是不是因为与background相反了呢?
在我标明的background信息中,已经说明了头发中能找到Mercury的存在
所以说A本身的假设就是个错误 因为Mercury根本不能eliminated.呢??


T,T求解答…………喵喵…………


24. The interstitial nucleus, a subregion of thebrain’s hypothalamus, is typically smaller for male cats than for female cats.A neurobiologist performed autopsies on male cats who died from disease X, adisease affecting no more than .05 percent of male cats, and found that thesemale cats had interstitial nuclei that were as large as those generally foundin female cats. Thus, the size of the interstitial nucleus determines whetheror not male cats can contract disease X.

Which one of the following statements, if true, mostseriously weakens the argument?

(D)Of 1,000 autopsies on male cats who did not contract disease X, 5 revealedinterstitial nuclei larger than those of the average male cat.
(E) The hypothalamus is known not to be causallylinked to disease Y, and disease X is a subtype of disease Y.

恩……还是先八一八偶滴思路:
【正常的】男猫’sH< 【正常的】女猫’s H
【得X病】男猫’s H>【正常的】女猫’s H
H导致→【得X病】
给出了两个事实 如何反驳?偶觉得应该举反例。。。
所以偶选了D o(╯□╰)o
D说明了【不得X病】男猫’s H也有>【正常的】女猫’s H


这里我最理解不能的是这个答案的解释:
E is the best answer.
Disease X is the subtype isdisease Y, if disease Y has no link with hypothalamus, X must have no link withhypothalamus.

X属于Y 。。YH无关。。X就要与H无关吗。。? 0 0
偶觉得这个实在是……………… 理解无能→ 比如猫科大部分爬树 狮子不爬树 难道就说明了狮子不属于猫科吗。。。
偶感觉偶的错误是不是sufficient/necessary弄反了…………但是实在转不过来这个弯
作者: yiayia    时间: 2012-3-27 23:18
第一题正向思维很难理解。
negative B:All the people in Beethoven’s time did ingest mercury.
如此的话,存在mercury不能证明贝多芬患有venereal disease,因为所有人都ingest mercury,贝多芬可能不是为了治病。

第二题答案的逻辑是这样的:
猫科都爬树,狮子是猫科,狮子爬树。
Y和H无关,X是Y的一种,X和H无关。
作者: babybearmm    时间: 2012-3-28 02:17
对了,这第10题我当时花了3'32'',当时完全没思路怎么prephrase,第一遍曾把所有选项都排除,然后推倒重来的。现在想想,我能不能这么理解?

In order to pressure the government of Country S tobecome less repressive, some legislators in Country R want to ban all exportsfrom R to S. Companies in R that manufacture telecommunication equipment
such as telephones and fax machines have argued that exports of their productsshould be exempted from the ban, on the ground that it is impossible for acountry to remain repressive when telecommunication
equipment is widely available to the population of that country.

Highlighted portion is the premise. premise是个条件命题,条件是"when telecommunication
equipment is widely available to the population of that country",那assumption,就得是说"when"这个前提条件成立。

(B) The telecommunication equipment that would be imported into S if the
exemption were to be granted would not be available solely to top government officials in S.



太精彩了!

第10题 D不是必要性假设,理解必要性假设,可以选用摘自sdacar的这一个比喻:
For the difference between sufficient assumption and necessargy assumption, here is an analogy:

Suppose you have city A and city B, separated by a river H.  If there are multiple bridges over the river H, each bridge is sufficient but not necessary to connect city A with city B. What is necessary is that at least one bridge over river H is working.

If there is only one bridge over the river H, then that bridge is both sufficient and necessary to connect city A with city B.

If there there are two rivers C and D separate the cities A and B, and there is one bridge over C and many bridges over D, then the bridge over C is necessary (but not sufficient) to connect city A and B, while all the individual bridge over D is neither necessary nor sufficient to connect city A with city B. What is necessary is that at least one bridge over D is working!
即使 negative D,  Of all exports that could be sent toCountry S, telecommunication equipment would not be the most effective in helpingcitizens of S oppose that country’s repressive government. 也不能否定结论。
What if it is the second or third effective?

14题,上邪自己尝试negative一下,如果negative 选项后,结论不成立,那么这就是必要性假设。
-- by 会员 yiayia (2012/3/26 22:38:55)





-- by 会员 babybearmm (2012/3/27 4:15:02)




-- by 会员 babybearmm (2012/3/27 4:50:03)

baby姐说的对,但全部排除掉的原因我认为是有些选项藏的太深,从反面看比较容易
必要性假设类negative就像比喻中的那样,把通过所必须的C桥给关闭了,那么肯定通不过了,也就是说结论不成立!
但如何negative是个技巧,需要练习。像B选项就是The telecommunication equipment that would be imported into S if the
exemption were to be granted would  be available solely to top government officials in S.
如果如上所述,on the ground when后面的条件不成立,结论也不成立。
正向思维有时候不如反向思维清晰。
-- by 会员 yiayia (2012/3/27 15:45:55)



谢谢yiayia,学到了!感觉我现在做CR有点局限性,就是总习惯于正向思考,prephrase答案,就像解数学题直接从题目出发严格计算出答案那样(话说我现在狂喜欢用解数学应用题的思路解CR)。这样,我第一遍看选项的时候,看到跟我想的方向不太对上眼的,就直接kill,然后就一不小心全都kill掉了。正像你说的“正向思维有时候不如反向思维清晰”,如果我看选项的时候用negation techniques去分析,就很容易选定B. 看来我要习惯于这种方法,扩宽思路

btw: 就说 "If A occurs, then B occurs" 这种命题取非就是 "exist a situation, in which A occurs but B does not occur"
这似乎是数学问题,数学上用的符号是:倒立的A (all)  --  镜像的E (exist)
作者: babybearmm    时间: 2012-3-28 02:57
historian那道题我跟你错得一样,当时一看答案就想“哎哟,可不是嘛!”感觉就是脑筋急转弯,可是做题的时候没有转过来。正像yiayia说的,这题就是反向思维,我现在还不习惯。

至于你的问题,A为啥错。我从逻辑数学角度说吧。比如以下这个命题:
“如果太阳从西边出来,那么yiayia是美女”  
别笑我荒唐,我这个命题可是永真命题
所谓“永真”,就是没人能反驳我。要反驳我这个命题,就必须得说:“存在某一天,当太阳从西边出来的时候,yiayia不是美女”
(就说 "If A occurs, then B occurs" 这种命题取非就是 "exist a situation, in which A occurs but B does not occur")
问题是,你能找到“某一天太阳从西边出来”的情况么?不能,谁也不能。
所以可怜的yiayia帅锅对我再咬牙切齿也不能反驳我
我可以说:“如果太阳从西边出来,那么__________”。后面空格随便填,任我扯得天花乱坠啼笑皆非,都是永真命题。
那如果你跟我辩论,说:babybearmm你说“太阳从西边出来”,这压根就是扯淡嘛,大错特错!
那么,你这样的发言,可以反驳“太阳从西边出来”,但不能削弱我的命题“如果太阳从西边出来,那么yiayia是美女”。

看这道题
Historian: If we find Hg in his hair, then we can conclude .....
(If 太阳从西边出来,then  yiayia是美女)
如果我们对A选项作取非验证,"some of Hg in the body could be eliminated",你认为那样头发中就有可能根本不存在Hg,这样的话"we find Hf in his hair"不成立。——问题在于,这不能否认Historian的命题,就像你说“太阳不从西边出来”这不能反驳我那个命题。

而正确答案B选项,如果取非: All people at that time ingested Hg.
那么这就能反驳Historian的if..then..命题:
If we find Hg in his hair, we CANNOT conclude ....

21. Historian:
background:We can learn about the medical history of individuals throughchemical analysis of their hair. It is likely, for example, that Isaac Newton’spsychological problems were due to mercury poisoning; traces of mercury werefound in his hair.
Analysis is now being done on a lock of Beethoven’shair. Although no convincing argument has shown that Beethoven ever had avenereal disease, some people hypothesize that venereal disease caused hisdeafness. Since mercury was commonly ingested in Beethoven’s time totreat venereal disease, if researchers find a trace of mercury in his hair, wecan conclude thatthis hypothesis is correct.

Which one of thefollowing is an assumption on which the historian’s argument depends?

(A)None of the mercury introduced into the body can be eliminated.
(B) Some people in Beethoven’s time did not ingestmercury.

这道题偶是这么分析的:
Mercury治愈venereal disease
Mercury存在于头发里
贝多芬头发含有Mercury 贝多芬得了venereal disease

我读完后的定向有两个:
1、 Mercury不会随着时间消失  消失了就检测不到这个病了也就说明不了什么问题了
2、 别的病不拿Mercury来治

后来对了答案后 理解的结果是:
Mercury不是一种普遍现象 如果吃Mercury是普遍现象那么也就不能说明venereal diseaseMercury的特殊性
但是感觉B的表述不是很清楚的说……因为some毕竟含有特例的因素在里面
那我也可以因此理解为:some特殊群体没事就吃Mercury。。跟我人民大众治病良方无关0 0
yiayia哥帮我解释一下这里词的用法好不好。。我感觉我对词的精准性一直理解地不是很到位 包括上一组的那个牙医的题 你一讲我突然就明白了那个inevitably & definitely 之前打死也想不通。。偶最近也在看bible。。不过还在理解中T.T 一直内化不了成自己的知识体系 求传授……5555……】

最后分析了一下自己错误的原因。。。感觉是不是因为与background相反了呢?
在我标明的background信息中,已经说明了头发中能找到Mercury的存在
所以说A本身的假设就是个错误 因为Mercury根本不能eliminated.呢??


T,T求解答…………喵喵…………


24. The interstitial nucleus, a subregion of thebrain’s hypothalamus, is typically smaller for male cats than for female cats.A neurobiologist performed autopsies on male cats who died from disease X, adisease affecting no more than .05 percent of male cats, and found that thesemale cats had interstitial nuclei that were as large as those generally foundin female cats. Thus, the size of the interstitial nucleus determines whetheror not male cats can contract disease X.

Which one of the following statements, if true, mostseriously weakens the argument?

(D)Of 1,000 autopsies on male cats who did not contract disease X, 5 revealedinterstitial nuclei larger than those of the average male cat.
(E) The hypothalamus is known not to be causallylinked to disease Y, and disease X is a subtype of disease Y.

恩……还是先八一八偶滴思路:
【正常的】男猫’sH< 【正常的】女猫’s H
【得X病】男猫’s H>【正常的】女猫’s H
H导致→【得X病】
给出了两个事实 如何反驳?偶觉得应该举反例。。。
所以偶选了D o(╯□╰)o
D说明了【不得X病】男猫’s H也有>【正常的】女猫’s H


这里我最理解不能的是这个答案的解释:
E is the best answer.
Disease X is the subtype isdisease Y, if disease Y has no link with hypothalamus, X must have no link withhypothalamus.

X属于Y 。。YH无关。。X就要与H无关吗。。? 0 0
偶觉得这个实在是……………… 理解无能→ 比如猫科大部分爬树 狮子不爬树 难道就说明了狮子不属于猫科吗。。。
偶感觉偶的错误是不是sufficient/necessary弄反了…………但是实在转不过来这个弯
-- by 会员 199249712 (2012/3/27 17:27:19)







作者: 199249712    时间: 2012-3-28 08:43
懂了!! baby姐姐的if太到位了
作者: 199249712    时间: 2012-3-28 08:46
第二个也懂了。。。我粗心了。比较错误。。。
作者: 199249712    时间: 2012-3-28 19:01
Questions49-50
Joseph: My encyclopedia saysthat the mathematician Pierre de Fermat died in 1665 without leaving behind anywritten proof for a theorem that he claimed nonetheless to have proved. Probably this alleged theorem simplycannot be proved, since---as the article points out---no one else has beenable to prove it. Therefore it is likely that Fermat was either lying or elsemistaken when he made his claim.
Laura: Your encyclopedia is outof date. Recently someone has in fact proved Fermat’s theorem. And since thetheorem is provable, your claim---that Fermat was lying or mistaken---clearlyis wrong.
50. Which one of the followingmost accurately describes a reasoning error in Laura’s argument?
(A) It purports to establishits conclusion by making a claim that, if true, would actually contradict thatconclusion.
(B) Itmistakenly assumes that the quality of a person’s character can legitimately betaken to guarantee the accuracy of the claims that person has made.
(C) It mistakes something that is necessary for itsconclusion to follow for something that ensures that the conclusion follows.
(D) It uses the term “provable”without defining it.
(E) It fails to distinguishbetween a true claim that has mistakenly between believed to be false and afalse claim that has mistakenly been believed to be true.








………………错误率好稳定啊 5/5/4

这道。。开始是属于完全没有思路的内种。。。 选项都太抽象啦

姐姐看这个结果正确吗。。。求确认。。求姐姐的思路。。。
A+B=C

something that ensures that the conclusion follows.


所以A+B=!C
B就成了
something that is necessary for its conclusion
so
It mistakes something that is necessary for itsconclusion to follow for something that ensures that the conclusion follows.

作者: babybearmm    时间: 2012-3-29 02:26
这题就是充分条件和必要条件。

Joseph:

Fermat died without any written proof
                                                                     } Fermat's theorem is NOT provable -----> Fermat was lying or making mistake (he alleged to have proved it)
no one has been able to prove


Laura:
someone recently proved ----> Fermat's theorem is provable ----> Fermat was NOT lying or .... (by claiming he had proved it)

You know, Laura's argument is unsound. By noting that someone is able to prove, we CANNOT conclude that Fermat himself was able to prove. Perhaps he was unable to prove, and he told a lie in claiming that he had proved it.

On the other hand, Joseph's argument is sound (I mean the last arrow, from Fermat's theorem is NOT provable to Fermat was lying or making mistake (he alleged to have proved it)), because if Fermat was honest, his theorem must be provable. Therefore, by noting that his theorem is not provable, Joseph concludes that Fermat was dishonest.

A = "Fermat's theorem is NOT provable"
B = "Fermat was lying or ..."

Joseph:      A----->B                      A是B的充分(sufficient)条件,B是A的必要(necessary)条件
Laura:        No A -----> No B

以上,抽象一下吧。就说一个命题"A--->B"(由A推出B),这里面:A是B的充分(sufficient)条件,B是A的必要(necessary)条件
逆命题是"B--->A",否命题是"no A ----> no B",逆否命题是"no B ---> no A"
一个命题和他的逆否命题,是等效的。
"A--->B" 等效于 "no B ---> no A"

那么上面,Joseph的"A--->B"能站住脚,而Laura提出的是这个命题的否命题"no A ----> no B",未必站得住脚。当且仅当A和B互为充要条件的时候,才能站住脚。

(btw:sorry后面这片我不知道怎么用英文说,我是小时候学奥数学的逻辑,用的中文。我是喜欢把那些长串英文用数学符号表示,就像解数学应用题设x, y variables一样,然后模式化推理,就像列方程解方程一样。这样比较准确快捷,因为不用通过英文长串语言去思考,就节省了大脑很多内存。)

Questions49-50
Joseph: My encyclopedia saysthat the mathematician Pierre de Fermat died in 1665 without leaving behind anywritten proof for a theorem that he claimed nonetheless to have proved. Probably this alleged theorem simplycannot be proved, since---as the article points out---no one else has beenable to prove it. Therefore it is likely that Fermat was either lying or elsemistaken when he made his claim.
Laura: Your encyclopedia is outof date. Recently someone has in fact proved Fermat’s theorem. And since thetheorem is provable, your claim---that Fermat was lying or mistaken---clearlyis wrong.
50. Which one of the followingmost accurately describes a reasoning error in Laura’s argument?
(A) It purports to establishits conclusion by making a claim that, if true, would actually contradict thatconclusion.
(B) Itmistakenly assumes that the quality of a person’s character can legitimately betaken to guarantee the accuracy of the claims that person has made.
(C) It mistakes something that is necessary for itsconclusion to follow for something that ensures that the conclusion follows.
(D) It uses the term “provable”without defining it.
(E) It fails to distinguishbetween a true claim that has mistakenly between believed to be false and afalse claim that has mistakenly been believed to be true.








………………错误率好稳定啊 5/5/4

这道。。开始是属于完全没有思路的内种。。。 选项都太抽象啦

姐姐看这个结果正确吗。。。求确认。。求姐姐的思路。。。
A+B=C

something that ensures that the conclusion follows.


所以A+B=!C
B就成了
something that is necessary for its conclusion
so
It mistakes something that is necessary for itsconclusion to follow for something that ensures that the conclusion follows.
-- by 会员 199249712 (2012/3/28 19:01:50)








作者: 199249712    时间: 2012-3-29 08:14
Probably this alleged theorem simplycannot be proved,

Therefore it is likely that Fermat was either lying or elsemistaken when he made his claim.

偶明白了…… 偶把结论弄错了…………谢谢姐姐~!
作者: babybearmm    时间: 2012-3-29 13:12
You got it!

Probably this alleged theorem simplycannot be proved,

Therefore it is likely that Fermat was either lying or elsemistaken when he made his claim.

偶明白了…… 偶把结论弄错了…………谢谢姐姐~!
-- by 会员 199249712 (2012/3/29 8:14:45)


作者: 199249712    时间: 2012-3-31 13:03


66.
A recent survey showed that many workers in acertain company are dissatisfiedwith their jobs. The survey also showed thatmost of the dissatisfied workersbelieve that they have little control overtheir job assignments. Therefore, toincrease workers’ job satisfaction thecompany’s management need onlyconcentrated on changing workers’ beliefsregarding the degree of control theyhave over their job assignment.
Which one of thefollowing, if also shown by the survey, would most seriouslycall into questionthe conclusion made by the author of the passage?
(A) The dissatisfied workers feel that theirwagesare too low and working conditions are unsatisfactory.

(D)Most people in company management believe that theworkers already have toomuch control over their work.


66. To be honest,我的直觉是A,然后还是花了4min来确认(主要是确定排除D)....
如果要说快捷的解题方法,我觉得就是抓住结论(相信直觉还是没错的):
Therefore, to increase workers job satisfactionthe company’s management need onlyconcentrate on changing workers’ beliefs regarding the degree of control theyhave over their job assignments.
In order to weaken the conclusion, we should show that there are other factors(independent of feeling self-controlled) that contribute to workers'satisfaction. Hence Choice A is the answer.

如果你要问我D为啥错,我还是借助心爱的数学玩具来解释吧:
Let Y be "worker's satisfaction" and X be "feeling self-controlled".Each of X and Y has a value of 0 or 1, with 0 being "no" and 1 being"yes". Set up a 2X2 matrix of joint probability p
xy:
 
Y=1



 
 
p01



 
 
p11



 
 
Y=0



 
 
p00



 
 
p10



 
 
joint probability



 
 
X=0



 
 
X=1



 

Now, translate all the english language to mathematical expressions:
many workers ... are dissatisfied with theirjobs
p
00+p10 islarge          (1)
The survey also showed that most of thedissatisfied workers believe that they have little control over their jobassignments.
p
00/p10> 1               (2)
D is saying:
(D)Most people in company management believe thatthe workers already have too much control over their work.
(p
10+p11)/(p00+p01)>1           (D)
note: 不好意思,我发现我在写这篇分析的时候,把D选项意思理解错了。我当时理解成了说most workers feel they already have self-control.还是有点区别的,这里说的不是对自己的感受评价,而是说对公司workers整体的感受评价。那么D选项就更加无关了。

As we know from statistics, "variables X and Y are positivelycorrelated" is equivalent to "the odds ratio is significantly largerthan 1". That is:
odds ratio = (p
00*p11)/(p01*p10)> 1            (*)
However, from what we know, i.e.(1) and (2),  we CANNOT conclude (*). Inother words, data could simply be  coincidence, NOT a manifestation ofcausal relationship ("not feeling self-controlled" leads to "nosatisfaction")
Also, (D) has NOTHING to do with (*). In other words, D neither strengthens norweakens the argument.

Also, there could be many other factors serving as determinants ofsatisfaction. In mathematical expression, Y is a function
(函数)of its determinants.
Y=f(X, A, B, C, ...)
or even without X:
Y=f(A, B, C, ...)

In summary, my points are:
(a) It is dubitable whether X is a determinant of Y;
(b) There are other determinants of Y, so X is NOT the ONLY determinant.
Therefore, Choice D is incorrect, and Choice A is correct by pointing out otherdeterminants.

btw: 我觉得这种joint probability matrix -- odds ratio 的统计学思路解CR题挺有用的,因为CR很多题就是研究variables之间是否存在相互联系。而likelyhood的数学表示就是odds ratio,这也会极大地简化咱的思维过程,让内在的逻辑暴露得更明显。一个类似的题目:
http://forum.chasedream.com/GMAT_CR/thread-695574-1-1.html




作者: 199249712    时间: 2012-3-31 13:05
再次狠狠的研究了一下姐姐的思路…… 懂了后觉得太赞了啊 尤其是kaplan的题……
~
!! 姐姐跟Yiayia哥绝对是大神啊。。

Choi: All other factors being equal, childrenwhose parents earned doctorates are more likely to earn a doctorate thanchildren whose parents who did not earn doctorates.

Harts: But consider this: Over 70 percent ofall doctorate holders do not have a parent who also holds a doctorate.

Which of the following is the most accurate evaluationof Hart's reply?

a. It establishes that Choi's claim is anexaggeration.
b. If true, it effecively demonstrates thatChoi's claim cannot be accurate.
c. It is consisitent with Choi's claim.
d. It provides alternative reasons foraccepting Choi's claim.
e. It mistakes what is necessary for an eventwith what is sufficient to determine that the event will occur.                                  




[attachimg]98343[/attachimg]

[attachimg]98344[/attachimg]

对应狒狒66题
作者: 199249712    时间: 2012-3-31 13:06

60. One of themore reliable methods of determining regional climatic conditionsinprehistoric periods is to examine plant pollen trapped in glacial iceduringancient times. By comparing such pollen samples with spores take frommodernvegetation, scientists can figure out approximately what the weather waslike atthe time of pollen deposition. Furthermore, by submitting theprehistoricsamples to radiocarbon dating techniques, we can also determine whencertainclimatic conditions were prevalent in that portion of the globe.
Which one of thefollowing may be inferred from the information in the passage?
(A) The earth hasundergone several glacial periods.
(B)Radiocarbon dating can be corroborated by glacialevidence.
(C) Similaritiesbetween prehistoric and contemporary climates do not exist.
(D) Pollen depositionis a fairly continuous process.
(E) Certain flora are reliably associatedwithparticular climatic conditions.






60.这题关键是读懂,就像scientific RC.
According to the stimulus, glacial evidence (i.e. the pollen samples) can tell what type of climate was at that time, and carbondating can tell "when certain climaticconditions were prevalent in that portion of the globe"
For example, with pollen samples, we can tell that at x million years ago(associated with the pollen sample), the climate is "hot and humid".And with carbon dating, we can tell that this particular type of climate("hot and humid") was prevalent in that place from x1 million yearsago to x2 million years ago.
Therefore, these two methods give different information. Hence Choice B iswrong.

Choice E is correct. Because scientists can tell the climate type from thepollen samples, we can safely infer that there are certain flowers associatedwith a particular climate type.
作者: 199249712    时间: 2012-3-31 13:06
59. Top college graduates are having moredifficulty demonstratingtheir superiority to prospectiveemployersthan did the top students of twenty years ago when an honors degreewasdistinction enough. Today’s employers are less impressed with the honorsdegree.Twenty years ago no more than 10 percent of a given class graduatedwith honors.Today, however, because of grade inflation, the honors degree goesto more than50 percent of a graduating class. Therefore, to restore confidencein thedegrees they award, colleges must take steps to control grade inflation.
Which one of the following is an assumption that, if true, would supporttheconclusion in the passage?
(A) Today’s studentsare not higher achieversthan the students of twenty years ago.
(B) Awarding too many honors degrees causescolleges toinflate grades.
(C) Today’s employers rely on honors ranking in making their hiringdecisions.
(D) It is not easy for students with low grades to obtain jobs.
(E) Colleges must make employers aware of the criteria used todetermine whoreceives an honor degree.




59这题我刚也做错了,错选了E。看答案发现,A是典型的defender assumption
B错是因为,题目说了"inflategrades ----> too many honors degree",因果关系是这样的,B把因果关系颠倒了,这属于out-of-scope,不能作为assumption. 另外"inflate grades ----> toomany honors degree"并不是论证的核心,而是cause of the cause.

那道SCAthat 能不能指代goal like的用法到底对不对呢
mm啊,一个压根就不对的句子,你问我这里面"that"指代什么,我咋说?永远别去justify错误句子的结构。
我就是从意思上判断,
the youth has this particular goal
is compared to
the earlier generations had this particular goal
所以必须用as啊,"like that"无论如何都不对,甭管that指什么。
作者: 199249712    时间: 2012-4-1 21:04
70. A low-pressure weather system is approaching Plainville; rainfall results from about 70 percent of such systems in the Plainville
area. Moreover, the current season, spring, is the time of year in which thundershowers, which sometimes result from low-pressure systems, are most likely to occur in Plainville.

Knowing which one of the following, in addition to the information above, would be most useful for determine the probability that Plainville will have a thundershower soon?


A.the percentage of thundershowers in Plainville that occur in the spring
B.the percentage of spring rainfalls in Plainville that are thundershowers ?
C.the percentage of thundershowers in Plainville that result from low-pressure systems
D.whether low-pressure systems in other areas are associated with rainfall
E.whether Plainville has more or fewer thundershowers each spring than do near by towns


额。。我的狒狒上写的这道题的答案是B
但是在解析上的答案是C


后来想了一下 觉得C是对的。。


我的思路是:
l/r=70
t/l=C
so t/r=70C

其中   l=low pressure
        t=thundershowers
        r=rainfall

我想错的主要原因是由于我们读错了第二句话的含义:
rainfall results from about 70 percent of such systems inthe Plainvillearea.
70%修饰的是low pressure
不是rainfall
所以比例是反的 所以可以相乘 相乘后就可以把分母消掉了

酱紫。。。
作者: yiayia    时间: 2012-4-1 23:35
We now see the only thing happening is that  a low-pressure weather system is approaching Plainville. It is the only information that we base on to make the inference.   Then the argument tells us two conditions. You have to figure out what is the meaning of "result from".
If we translate it into the maths words, it should be presented in this way:
l=low pressure
t=thundershowers
r=rainfall
That is P(r)/P(l)=70% if l happens, the probability of r is 70%. The stimulus asks us p(t)/p(l)=?
B tell us that P(t)/p(r)=?  
So if we multiply these two percentage, we actually can judge the probability that Plainville will have a thundershower.
Let's check the answer C. I have read the reasoning in the feifei logic. It is totally wrong. C told that P(l)/P(t)=?  However,we cannot simply reverse the percentage. For example,   P(l)/P(t)= 50%, and then the reversed percentage P(t)/P(l)= 200%. How can it be possible?

If we jump out of the scope of "maths", we can eliminate C because there maybe other reasons for a thundershower. We can say the probability that Plainville will have a thundershower soon after knowing a low-pressure weather system is 100%. But because of others causes, the thundershowers are not all caused by  low-pressure weather systems. Thus the percentage of thundershowers in Plainville that result from low-pressure systems is 50%. As far as we can see, this choice is not "strict" enough.

The right answer is B. You have done a great job, and just believe yourself.
作者: babybearmm    时间: 2012-4-2 03:25
Just add my 2c
This question is about conditional probability (条件概率)。
Question: p(R|L)=70%, p(T|L)=?
Choice B: p(R|T) is known
Choice C: p(L|T) is known
....

I like yiayia's intuitional thought:
"For example,   P(l)/P(t)= 50%, and then the reversed percentage P(t)/P(l)= 200%. How can it be possible?"
"If we jump out of the scope of "maths", we can eliminate C because there maybe other reasons for a thundershower. "
作者: 199249712    时间: 2012-4-2 14:58
GWD-10-Q30:GWD-2-17
Thequality of unrefined olive oil is not actually defined in terms of acidity, yet extensive testshave shown
that the less free oleic acid an unrefined olive oil contains perliter, the higher its quality.  
Theproportion of free oleic acid that an olive oil containsis an accurate measure of the oil’s acidity.


If the statements aboveare all true, which of the following conclusions is best supported by them?

A.     When an olive oil is refined, the concentrationof oleic acid in the oil is reduced.
B.     The quality of an unrefined olive oil can be determinedonly by accurately measuring its acidity.
C.      If an unrefined olive oil is intermediate in aciditybetween two other unrefined olive oils, it will also be intermediate betweenthem in quality.
D.     Free oleic acid is the only acid that unrefined olive oilcontains.
E.      People who judge the quality of unrefined olive oilsactually judge those oils by their acidity, which the judges can taste.




一道诡异的争议题0 0

偶感觉是A耶。。。 因为C说的酸不是free acidity啊。。。
我中和酸性也可能把别的酸掺进来了。。。

但是A也有问题 A说的是 an olive oil is refined
题目说的是 unrefined olive

T.T

作者: 199249712    时间: 2012-4-2 15:00
关于因果型//条件型结论
作者: babybearmm    时间: 2012-4-2 16:04
因为C说的酸不是free acidity啊。。。
The stimulus says, "The proportion of free oleic acid that an olive oil contains is an accurate measure of the oil’s acidity. "
In mathematical term, it means that:
(oil's acidity) = k * (% of free oleic acid)
, in which k is the coefficient.

"我中和酸性也可能把别的酸掺进来了。。。"
你理解错了,题目没有涉及任何酸碱中和反应

“但是A也有问题 A说的是 an olive oil is refined 题目说的是 unrefined olive
Exactly!

GWD-10-Q30:GWD-2-17
Thequality of unrefined olive oil is not actually defined in terms of acidity, yet extensive testshave shown
that the less free oleic acid an unrefined olive oil contains perliter, the higher its quality.  
Theproportion of free oleic acid that an olive oil containsis an accurate measure of the oil’s acidity.


If the statements aboveare all true, which of the following conclusions is best supported by them?

A.     When an olive oil is refined, the concentrationof oleic acid in the oil is reduced.
B.     The quality of an unrefined olive oil can be determinedonly by accurately measuring its acidity.
C.      If an unrefined olive oil is intermediate in aciditybetween two other unrefined olive oils, it will also be intermediate betweenthem in quality.
D.     Free oleic acid is the only acid that unrefined olive oilcontains.
E.      People who judge the quality of unrefined olive oilsactually judge those oils by their acidity, which the judges can taste.




一道诡异的争议题0 0

偶感觉是A耶。。。 因为C说的酸不是free acidity啊。。。
我中和酸性也可能把别的酸掺进来了。。。

但是A也有问题 A说的是 an olive oil is refined
题目说的是 unrefined olive

T.T
-- by 会员 199249712 (2012/4/2 14:58:29)


作者: 199249712    时间: 2012-4-2 16:23
谢谢姐姐 我明白了………… 我怕你要睡了没敢信息你 结果姐姐来看偶帖子了。。
你那很晚了啊。。。 好刻苦。。~!!
作者: 199249712    时间: 2012-4-6 11:50
T-3-Q32.
A theory is either true or false. Galileo’sobservations of Jupiter’s satellites showed that the Ptolemaic theory of themotion of celestial bodies is false. Therefore, since the Copernicantheory of planetary motion is inconsistent with the Ptolemaicaccount, Galileo’s observations of Jupiter’s satellites proved the truth of theCopernican theory.


The argument above is open to the objection that itmakes the questionable assumption that


A.     Whoever first observed something inconsistent with thetruth of the Ptolemaic theory should be credited with having proved that theoryfalse.

B.     There are some possible observations thatwould be inconsistent with the account given by the Copernican theory butconsistent with the account given by the Ptolemaic theory.

C.     The Ptolemaic and Copernican theories,being inconsistent, cannot both be based on exactly the same evidence

D.     Numerous counterexamples were necessary inorder to show the Ptolemaic theory to be false

E.       The Ptolemaic and Copernican theories,being inconsistent, cannot both be false.





偶是一个粗心的孩子。。。偶为什么没看见first呢 = =

作者: babybearmm    时间: 2012-4-6 12:27
This is a good example of the logical error called "false dilemma".
作者: 199249712    时间: 2012-4-21 20:43
One of the most effective techniques for quickly finding correct answer choices and avoiding incorrect answer choices is prephrasing. Prephrasing an answer involves quickly speculating on what you expect the correct answer will be based on the information in the stimulus.
作者: 199249712    时间: 2012-4-22 17:37
艾玛从一个自习室被赶到另一个自习室- -|||烦死了都
1、刷了25到CR
2、做了阅读小分队
3、看陈向东数学。。。看了34页吧。。。- - 全是小学奥数题。。作为一个深恶痛绝奥数且偏科的孩子 我表示数学很无力
4、做做语法。。看看错题。。。

未完成:
1、看bible
- -完全不想看bible。。。
搜点新文章来看 不知道GMAT的阅读速度要求是多少啊??
作者: 199249712    时间: 2012-4-22 21:30
转载于economics。。。偶没有记住地址 随便点的
The science of civil war
         What makes heroic strife      Computer models that can predict the outbreak and spread of civil conflict are being developedFOR the past decade or so, generals commanding the world’s most advanced armies have been able to rely on accurate forecasts of the outcomes of conventional battles. Given data on weather and terrain, and the combatants’ numbers, weaponry, positions, training and level of morale, computer programs such as the Tactical Numerical Deterministic Model, designed by the Dupuy Institute in Washington, DC, can predict who will win, how quickly and with how many casualties.
Guerrilla warfare, however, is harder to model than open battle of this sort, and the civil insurrection that often precedes it is harder still. Which, from the generals’ point of view, is a pity, because such conflict is the dominant form of strife these days. The reason for the difficulty is that the fuel of popular uprisings is not hardware, but social factors of a type that computer programmers find it difficult to capture in their algorithms. Analysing the emotional temperature of postings on Facebook and Twitter, or the telephone traffic between groups of villages, is always going to be a harder task than analysing physics-based data like a tank’s firing range or an army’s stocks of ammunition and fuel.


Harder, but not impossible. For in the war-games rooms and think-tanks of the rich world’s military powers, bright minds are working on the problem of how to model insurrection and irregular warfare. Slowly but surely they are succeeding, and in the process they are helping politicians and armies to a better understanding of the nature of rebellion.
SCARE tactics
One of the best-known projects in this field is SCARE, the Spatio-Cultural Abductive Reasoning Engine, developed at the United States Military Academy at West Point by a team led by Major Paulo Shakarian, a computer-scientist-turned-soldier. SCARE operates at the most militarily conventional end of the irregular-conflict spectrum: the point where an army of guerrillas is already in being and is making life hard for a notionally better-armed army of regular troops. That, of course, has been the experience of American forces in Vietnam, Iraq and Afghanistan. Major Shakarian and his team have analysed the behaviour of guerrillas in both Iraq and Afghanistan, and think they understand it well enough to build reliable models.


Their crucial insight is the local nature of conflict in these countries. In particular, bombs directed at occupying forces are generally planted close to the place where they were made, and on the territory of the bombmaker’s tribal kin or co-religionists. That is not a surprise, of course. Kin and co-religionists are the most reliable allies in wars where different guerrilla groups may not always see eye to eye about objectives, beyond the immediate one of driving out foreign troops. But it does give Major Shakarian and his team a convenient way in. Using the co-ordinates of previously bombed sites, data from topographical and street maps, and information on an area’s ethnic, linguistic and confessional “human terrain”, SCARE is able to predict where guerrillas’ munition dumps will be to within about 700 metres. That is not perfect, but it is close enough to be able to focus a search in a useful way.
Moreover, SCARE’s focus should soon become more precise. Major Shakarian’s latest trick is to include data on phone-traffic patterns in the calculations. An upgraded version of the program, employing this trick, will be created next month.
All of which is useful for dealing with a conflict once it has started. But it is better, if possible, to see what may happen before things get going. And for that, America’s navy has a project called RiftLand.
RiftLand is being developed on the navy’s behalf by Claudio Cioffi-Revilla, a professor of computational social science at George Mason University in Virginia. It is specific to the part of East Africa around the Great Rift Valley (hence the name). That this area includes Congo, Ethiopia, Rwanda, Somalia and Uganda, each of which has been the scene of present or recent civil strife, is no coincidence. But the ideas involved could be generalised to other parts of the world, with due alteration for local conditions.


Broadly, RiftLand works by chewing its way through a range of data collected by charities, academics and government agencies, and uses these to predict where groups of people will go and with whom they may clash in times of drought or armed conflict. Dr Cioffi-Revilla gives the example (though he will not name names specifically) of a tribe of nomadic herders known for sharing its notions of veterinary medicine with others. This tribe, the model predicts, will reckon it safer to cross the lands of groups who also rely on keeping their animals healthy. Another point is that tribes who own a radio or mobile phone will steer clear of roads after news reports of government atrocities against their kin. A third is that much of the movement of herdsmen can be predicted from satellite data on the condition of pasture lands, modified by knowledge of what Dr Cioffi-Revilla calls “the complex network of IOUs” between tribes: which are currently hostile to one another, and who owes whom favours.


Hostile sentiments
The sort of conflict dealt with by RiftLand—a war of all against all in countries where central government is light or non-existent—has been particularly characteristic of this part of Africa in recent years. Further north, where states are stronger, urban insurrection of the sort seen at the beginning of the Arab spring is a more common threat. Politicians faced with such uprisings may thus be interested in yet another piece of software, known as Condor, which has been developed by Peter Gloor of the Massachusetts Institute of Technology. Dr Gloor is certainly not in the business of saving the jobs of Middle-Eastern dictators. He is actually a consultant to the Christian Democratic Union, Germany’s largest political party. But all politicians in power, whether democrats or dictators, share a distaste for demonstrations and protests on the streets.
Condor works by sifting through data from Twitter, Facebook and other social media, and using them to predict how a public protest will evolve. It does so by performing what Dr Gloor calls “sentiment analysis” on the data.
Sentiment analysis first classifies protesters by their clout. An influential Twitter user, for instance, is one who has many followers but follows few people himself. His tweets are typically upbeat (containing words or phrases such as “great”, “fun”, “funny”, “good time”, “hilarious movie”, “you’ll love” and so forth), are rapidly retweeted, and appear to sway others. In a nod to the methods developed by Google, Dr Gloor refers to this process as “PageRanking for people”.
Having thus ranked protesters, Condor then follows those at the top of the list to see how their output changes. Dr Gloor has found that, in Western countries at least, non-violent protest movements begin to burn out when the upbeat tweets turn negative, with “not”, “never”, “lame”, “I hate”, “idiot” and so on becoming more frequent. Abundant complaints about idiots in the government or in an ideologically opposed group are a good signal of a movement’s decline. Complaints about idiots in one’s own movement or such infelicities as the theft of beer by a fellow demonstrator suggest the whole thing is almost over.
Condor, then, is good at forecasting the course of existing protests. Even better, from the politicians’ point of view, would be to predict such protests before they occur. Not surprisingly, several groups of researchers are trying to do this too.
 
             
Aptima, a firm based in Woburn, Massachusetts, is one. Its program, called E-MEME (Epidemiological Modelling of the Evolution of MEssages) uses sentiment analysis to see how opinions and states of mind flow across entire populations, not just activists. It employs data from online news sources, blogs and Twitter, and attempts to rank the “susceptibility” of certain parts of the populace to specific ideas. According to Robert McCormack, the project's chief technologist, E-MEME can determine things as different as which places in Egypt contain people who will care a lot about a border incident with Israel, and which parts of a country most need water in times of drought.
The Worldwide Integrated Crisis Early Warning System (W-ICEWS) project, led by Lockheed Martin, a large American defence contractor, goes even further. According to Lieutenant-Colonel Melinda Morgan of the office of the secretary of defence, in Washington, who is the government’s liaison officer for the project, it can crunch great quantities of data from digital news media, blogs and other websites, and also intelligence and diplomatic reports. It then uses all this to forecast—months in advance—riots, rebellions, coups, economic crises, government crackdowns and international wars. Colonel Morgan calls this process “social radar”.
Conflict forecasters are even joining the open-source bandwagon, in an attempt to improve their software. Last August IARPA, an American-government technology-development agency for the intelligence services, started the Open Source Indicators programme. This finances developers of software that can “beat the news”: forecasting political crises and mass violence in a reliable way. The programme’s manager, Jason Matheny, is now considering the proposals that have come in so far. These range from tracking Wikipedia edits to monitoring traffic with roadside cameras. The only proposals Mr Matheny will not consider are those designed to forecast conflict in America itself (the CIA is not supposed to spy on people in the United States), and those that rely on monitoring particular individuals, whether in America or elsewhere.
Guerrillas in the midst
Rather than just foretelling the future, however, the best technology should concentrate on shaping it. W-ICEWS offers a bit of that. It has a “what if” capability, which allows users to change the inputs and see how things might develop differently given different events in the real world. But Venkatramana Subrahmanian of the University of Maryland proposes something more specific. The Temporal-Probabilistic Rule System, a program his team has developed using $600,000 of American-army money, looks at 770 social and political indicators and uses them to predict attacks by Lashkar-e-Taiba, a guerrilla group based in Pakistan-administered Kashmir. If it works, this process might be applied, using a different set of indicators, to other groups of rebels.
The crucial point about Dr Subrahmanian’s model is that it not only predicts attacks, it also suggests how they might be countered. Dr Subrahmanian is understandably cagey about the details, but he does give one example: if an attack requires complex co-ordination between group members, the software might recommend “stoking paranoia” by forging false communications between them.
On April 2nd President Barack Obama announced a $10m bounty on Lashkar-e-Taiba’s leader, Hafiz Saeed. It would indeed mark the coming of age of civil-strife software if that bounty, or another like it, were one day claimed on behalf of a group of programmers half a world away.

250 1;26
268 1:26

329 2:09
251 1:33
251 1:38
216 1:29

225 1:28
作者: 199249712    时间: 2012-4-22 21:33
Manufacturing          The third industrial revolution      The digitisation of manufacturing will transform the way goods are made—and change the politics of jobs too

THE first industrial revolution began in Britain in the late 18th century, with the mechanisation of the textile industry. Tasks previously done laboriously by hand in hundreds of weavers’ cottages were brought together in a single cotton mill, and the factory was born. The second industrial revolution came in the early 20th century, when Henry Ford mastered the moving assembly line and ushered in the age of mass production. The first two industrial revolutions made people richer and more urban. Now a third revolution is under way. Manufacturing is going digital. As this week’s special report argues, this could change not just business, but much else besides.
A number of remarkable technologies are converging: clever software, novel materials, more dexterous robots, new processes (notably three-dimensional printing) and a whole range of web-based services. The factory of the past was based on cranking out zillions of identical products: Ford famously said that car-buyers could have any colour they liked, as long as it was black. But the cost of producing much smaller batches of a wider variety, with each product tailored precisely to each customer’s whims, is falling. The factory of the future will focus on mass customisation—and may look more like those weavers’ cottages than Ford’s assembly line.
Towards a third dimension
The old way of making things involved taking lots of parts and screwing or welding them together. Now a product can be designed on a computer and “printed” on a 3D printer, which creates a solid object by building up successive layers of material. The digital design can be tweaked with a few mouseclicks. The 3D printer can run unattended, and can make many things which are too complex for a traditional factory to handle. In time, these amazing machines may be able to make almost anything, anywhere—from your garage to an African village.
The applications of 3D printing are especially mind-boggling. Already, hearing aids and high-tech parts of military jets are being printed in customised shapes. The geography of supply chains will change. An engineer working in the middle of a desert who finds he lacks a certain tool no longer has to have it delivered from the nearest city. He can simply download the design and print it. The days when projects ground to a halt for want of a piece of kit, or when customers complained that they could no longer find spare parts for things they had bought, will one day seem quaint.
Other changes are nearly as momentous. New materials are lighter, stronger and more durable than the old ones. Carbon fibre is replacing steel and aluminium in products ranging from aeroplanes to mountain bikes. New techniques let engineers shape objects at a tiny scale. Nanotechnology is giving products enhanced features, such as bandages that help heal cuts, engines that run more efficiently and crockery that cleans more easily. Genetically engineered viruses are being developed to make items such as batteries. And with the internet allowing ever more designers to collaborate on new products, the barriers to entry are falling. Ford needed heaps of capital to build his colossal River Rouge factory; his modern equivalent can start with little besides a laptop and a hunger to invent.
Like all revolutions, this one will be disruptive. Digital technology has already rocked the media and retailing industries, just as cotton mills crushed hand looms and the Model T put farriers out of work. Many people will look at the factories of the future and shudder. They will not be full of grimy machines manned by men in oily overalls. Many will be squeaky clean—and almost deserted. Some carmakers already produce twice as many vehicles per employee as they did only a decade or so ago. Most jobs will not be on the factory floor but in the offices nearby, which will be full of designers, engineers, IT specialists, logistics experts, marketing staff and other professionals. The manufacturing jobs of the future will require more skills. Many dull, repetitive tasks will become obsolete: you no longer need riveters when a product has no rivets.
The revolution will affect not only how things are made, but where. Factories used to move to low-wage countries to curb labour costs. But labour costs are growing less and less important: a $499 first-generation iPad included only about $33 of manufacturing labour, of which the final assembly in China accounted for just $8. Offshore production is increasingly moving back to rich countries not because Chinese wages are rising, but because companies now want to be closer to their customers so that they can respond more quickly to changes in demand. And some products are so sophisticated that it helps to have the people who design them and the people who make them in the same place. The Boston Consulting Group reckons that in areas such as transport, computers, fabricated metals and machinery, 10-30% of the goods that America now imports from China could be made at home by 2020, boosting American output by $20 billion-55 billion a year.
The shock of the new
Consumers will have little difficulty adapting to the new age of better products, swiftly delivered. Governments, however, may find it harder. Their instinct is to protect industries and companies that already exist, not the upstarts that would destroy them. They shower old factories with subsidies and bully bosses who want to move production abroad. They spend billions backing the new technologies which they, in their wisdom, think will prevail. And they cling to a romantic belief that manufacturing is superior to services, let alone finance.
None of this makes sense. The lines between manufacturing and services are blurring. Rolls-Royce no longer sells jet engines; it sells the hours that each engine is actually thrusting an aeroplane through the sky. Governments have always been lousy at picking winners, and they are likely to become more so, as legions of entrepreneurs and tinkerers swap designs online, turn them into products at home and market them globally from a garage. As the revolution rages, governments should stick to the basics: better schools for a skilled workforce, clear rules and a level playing field for enterprises of all kinds. Leave the rest to the revolutionaries.
作者: 199249712    时间: 2012-4-22 21:34
[attachimg]99415[/attachimg]
作者: yiayia    时间: 2012-4-22 22:11
孩子 悠着点。。 1周10篇很容易看完的。。
作者: 199249712    时间: 2012-4-22 22:52
yiayia哥我现在处于暴力阶段 嘻嘻。。。悠着悠着就3战了 你说10篇啥。。?
作者: 199249712    时间: 2012-4-23 00:58
转载于NYT。。。 也是首页随便点的。。。
THE complimentary wine and fruit platter was sent up to Jessica Griffin and her family moments after they strolled into their roomy suite. They were accompanied by a bellhop who placed their bags near a tidy crib made up with luxurious, high thread-count sheets for Ms. Griffin’s 1-year-old daughter.                Enlarge This Image

Fred R. Conrad/The New York TimesAgent Emily Meredith Prentiss secures exclusive rates and special treatment for her clients. Still, the travel tip she offers is simple: plan ahead for better rates.                            

 
           Readers’ Comments                            
Share your travel agent tips and experiences.
                                           

The V.I.P. treatment at the Cheeca Lodge and Spa in the Florida Keys last month hadn’t come with an extra cost. In fact, Ms. Griffin said, she paid about $100 a night less than the standard rate for her room. And the deal wasn’t the result of hours of tedious online research either. She had finagled her savings the old-fashioned way: through a travel agent.        
“I needed recommendations and someone to steer me in the right direction,” said Ms. Griffin, who opted to work with an agent after years of making her own reservations because she needed a getaway suitable for a toddler and had little interest in scrolling through endless and conflicting user hotel reviews online. “There are so many,” she said. And with every site displaying beautiful pictures and tantalizing offers, “it can be overwhelming.”        
“I wanted somebody from a reputable agency who could say yes, you’ll enjoy this stay,” she said.        
According to those in the travel agent industry, clients like Ms. Griffin are not alone, and are in fact helping to stanch the bloodletting the industry has experienced since the onset of D.I.Y. booking more than a decade ago. Nearly one in three leisure agencies is hiring, according to PhoCusWright, a travel research firm. And in 2011 travel agencies experienced a second consecutive year of growth; their bookings account for a third of the $284 billion United States travel market.        
This comes after years during which all signs seemed to be suggesting that travel agents would soon go the way of telex operators. And it’s true that the numbers are stark: During the industry’s peak years of the mid-1990s, there were about 34,000 retail locations booking trips. Today, there are 14,000 to 15,000, according to PhoCusWright. In 2009 alone, in the throes of the recession, bookings through traditional agencies plummeted by 23 percent.        
But now, some green shoots. An improving economy and the corporate travel that goes with it seem to be converging with a population for whom booking travel online has become increasingly onerous and time-consuming. Just how time-consuming? Steve Peterson, the global travel and transportation leader for the I.B.M. Institute for Business Value, set out to answer that very question. In a survey of more than 2,000 travelers worldwide, 20 percent said it took them more than five hours to search and book travel online. Nearly half said it required more than two hours.        
No one expects agency business to rebound to pre-Internet levels, but recent signs — like the fact that leisure travelers accounted for a 10 percent bump in sales in 2010 (a bit less in 2011) — suggest that agents can still play a relevant role. And though no one has been keeping track of the reasons travelers are turning to actual human beings, Mr. Peterson suspected it might have something to do with the drawbacks of the Web. “It’s come to a point that it’s too much information to be confident that they have the ability to book the lowest fare,” or uncover the best place to stay, he said of the respondents. “Consumers are hungry for that one-and-done shopping experience.”        
As it turns out, after years of losing ground to online sites, a new breed of tech savvy, specialized and collaborative agent has emerged.
作者: babybearmm    时间: 2012-4-23 04:41
wow~ this thread seems to become your diary! nice~
keep going.

i think yiayia meant that a non-subscriber has limited access (10 articles per month)
but don't worry, even without subscription there are a bunch of valuable stuffs on the web (I can name a few great websites), and you can turn to me when you need an article (i am a subscriber to economist).
作者: yiayia    时间: 2012-4-23 08:21
yiayia哥我现在处于暴力阶段 嘻嘻。。。悠着悠着就3战了 你说10篇啥。。?
-- by 会员 199249712 (2012/4/22 22:52:51)

the economist 网站 1周只能看10篇
作者: 199249712    时间: 2012-4-23 12:10
哇真的吗。。谢谢姐姐~!!!! 太感谢了mua~!!!
作者: 199249712    时间: 2012-4-23 12:10
昨天周天 早知道先下10篇再说了
作者: 199249712    时间: 2012-4-23 21:45
艾玛今天就各种上课 晚上总结了十道逻辑题。。。
白天看了一篇1700的文章。。。 我要逃课。。。
作者: 199249712    时间: 2012-4-24 18:40
转自ecomics

Decision making in cricket        In the blink of an eye  IT IS described by those who witnessed it as the greatest over ever bowled: Michael Holding, taking the new ball for the West Indies in the 1981 Test against England at the Kensington Oval in Barbados. They called him “whispering death”. His run-up, which started close to the sight-screen, was so graceful, the feet so light upon the turf, that it was said that the umpires couldn’t hear him approach. The action was beautifully languid, so that he appeared to put in no effort into the delivery. And yet the ball would whistle past the batsman’s nose at unplayable speed. At a time when the West Indies had a quartet of fast bowlers considered to be the most fearsome the world had ever seen, Mr Holding was perhaps the slipperiest.
That day he was at his terrifying peak. Geoffrey Boycott, the unfortunate batsman at the other end, said it was the fastest over he had ever faced. And he was no yellow youth. He was one of England’s greatest-ever batsman, with a wonderful eye and an unflappable temperament. Yet for those six balls he played as if he were a blind man swatting at swifts. They say of a sniper’s bullet that it if you hear it, then you are safe, because it will already have passed safely by. It is the ones that you don’t hear that do for you. In that over, five balls screeched past Mr Boycott’s flailing bat. The sixth made a horrible mess of his stumps.
For the mortals in the stands, it is perhaps the greatest mystery in the game: how can top batsmen play fast bowling? That day Mr Holding was clearly too good for Mr Boycott. But in the next Test, at the Antigua Recreation Ground, he scored a century against the same bowling attack.
The generally-accepted definition of a fast bowler is one who bowls consistently at over 90 miles (145 km) per hour. A cricket pitch is just 22 yards (20 metres) long. That gives batsmen around half a second to pick up the trajectory of the ball, decide which shot to play, and then to execute it. As if that were not hard enough, the best bowlers not only get the ball to swing laterally through the air, but also to deviate off the pitch. In that time the batsman must make up his mind whether to defend the ball, attack it, leave it to pass the stumps or, if it is aimed at his chin, take evasive action.
When playing a cross-batted shot, such as a pull or a cut, the timing needed to connect with the ball seems impossible. According to a study in Nature Neuroscience, “the batsman must judge the vertical position of the ball to within 3cm (limited by the bat's width) and its time of arrival to within 3 milliseconds (limited by the time the ball takes to pass the effective percussion zone of the bat).”
So what sets such batsmen apart? It is tempting to assume that they simply have better visual reaction times than the rest of us and can pick the ball up quicker. But according to “Wait”, a new book by Frank Partnoy, that is not the case. The book is about general decision-making in life, but contains a chapter on “super-fast sports”. It concludes that the best batsmen are no faster at “seeing” than their less successful colleagues, or even many amateurs. Whether you are Virender Sehwag or a village-green clubber, it will take you around 200 milliseconds to react to the ball. The best batsmen are set apart by what happens in the next 200 milliseconds, which the book calls the preparation stage. This means deciding on the shot, moving into the correct position and swinging the bat. (The third stage, hitting the ball, accounts for the last 100 milliseconds.) And here the margin between us and them is miniscule: “A cricket batsman who is just fifty milliseconds slower than an average professional—in other words, someone who is slower by just a fraction of the time it takes to blink—simply has no chance of competing with the pros.” Quoting Peter McLeod, an Oxford professor, the book goes on: “Their skill, it seems, lies in how they use the information to control motor actions once they have picked it up, not in the more elementary process of picking it up.”
This cannot be done consciously, because conscious contemplation takes at least half a second, by which time your stumps may have been uprooted. Few sports can match cricket's need for super-fast, unconscious reaction. Baseball is one; tennis—particularly returning serve—is another. The fastest reaction times of all, claims Mr Partnoy, are needed in fencing. To score an épée, he claims, you must beat your opponent to the hit by just 40 milliseconds. By comparison, players in football, American football and basketball have an eternity to weigh their options.
Now a commentator, Mr Boycott is often asked his advice on how to play fast bowling. He has a simple mantra: watch the ball and play it as late as possible. Of course, he never thought about the mechanics of what it means to judge a stroke to within the time it takes to blink an eye. To him, as with all great players, it was mere instinct (honed, naturally, by dedicated training). For the rest of us mortals, perhaps the only consolation is that we have always been just 50 milliseconds away from greatness.

表示郁闷 我为什么要摘这篇- -、、、
作者: 199249712    时间: 2012-4-24 18:40
Facebook's flotation        Revision time  

ANYONE who thought that Facebook would run out of steam ahead of its initial public offering (IPO), which is expected to take place next month, should think again. On April 23rd the company revealed in a revised pre-IPO filing that it now boasts some 901m monthly active users, up from 845m in December 2011. If it continues to grow at this rate, the social-networking behemoth could soon boast one billion users, or around one in seven people on the planet.
But can Facebook translate this momentum into steadily growing profits against the backdrop of a volatile world economy and a growing battle over software patents that is roiling the tech industry? The social network is already locked in a fierce legal tussle with Yahoo!, which has accused it of violating a number of patents it holds in areas such as privacy controls and online advertising. And chances are that Facebook will be hit with more such lawsuits in future as its success makes it a tempting target.
This explains why the company has decided to fork out $550m on 650 patents held by Microsoft, which owns a small stake in Facebook. The social network will add these to the 750 patents that it snapped up from IBM for an undisclosed amount in March. The latest deal will enable Microsoft to recoup some of the $1.1 billion that it handed over to AOL earlier this month for a treasure trove of patents and patent applications, whilst retaining the rights to use the ones that most interest it. (Some commentators also see this as strengthening Microsoft and Facebook's united front against competitors such as Google.)
Experts think similar coalitions between tech firms will become more common in future because by working together they can afford to buy larger portfolios of patents and also compete more effectively against what is known as “patent trolls”, outfits that specialise in acquiring patents to make money from them purely through licensing or litigation in the courts. Frank Azzopardi of Davis Polk & Wardwell, a law firm, says the amount of collaboration around patent purchases has reached unprecedented levels. Earlier this year, for instance, another coalition led by Microsoft and Apple received a green light from American anti-trust regulators to buy a big portfolio of patents from Nortel Networks, a Canadian firm.
As well as strengthening Facebook's patent portfolio, its bosses must also demonstrate that they can keep its revenues and profits heading in the right direction. In the first quarter of this year, the company clocked up just over $1 billion of revenue, a 45% increase over the same period of 2011. But some analysts had been hoping for more.  In its regulatory filing the company said that average revenue per user had fallen 12% compared with the fourth quarter of the previous year, but said that it was common to see sales dip after a busy holiday season.
True, but Facebook recently spent a cool billion dollars in cash and stock on Instagram, a photo-sharing service. And potential investors will want reassurance that it can make handsome returns on their money. For now, the company seems set on investing heavily: its net income fell nearly a third from the fourth quarter of 2011 to $205m in the first three months of this year as expenses soared to $677m from $343 million a year earlier. Pouring money into new data centres, new hires and other areas makes sense ahead of a flotation. But once Facebook has gone public, its new investors may well expect it to pile up profits as well as patents.
作者: 199249712    时间: 2012-4-24 18:40
来写一下今天的完成的任务
早上阅读小分队  下午看逻辑大全 做了40道 背单词 总结OGcr 10道。。
然后就这个点了。。。

9点半了。。。GWD数学+语文
12点54.。刷到OG85题。不困但看不进去了。。。
背会单词睡咯。。喵喵喵
作者: babybearmm    时间: 2012-4-25 06:27
上邪加油~~
作者: 199249712    时间: 2012-4-25 08:39
谢谢姐姐~~~
作者: 199249712    时间: 2012-4-25 16:01
来写一下。。。被CR大全完虐 早上一个小时+ 下午2点到4点 都在做大全 才做了一套20道。。 错了一半 查了查讨论帖 很多答案不太认同。。。搁置了。 艾玛艾玛艾玛 CR啊。。救命啊。。。
19:38  总结Prep20道+吃饭。。。
21:39.。。。总结了倒一倒二两篇RC (*……(……)(&……%¥*&%&*(……(*……* 今天效率好低 做个小分队的速度就可以回寝室了
21:54 两个速度阅读加记录。。。。啊我饿了!!!
23:35 总结1篇阅读
12:22 总结CR5道。。。。。。。。。。。。。。。。。。。困。。晚安咯
作者: babybearmm    时间: 2012-4-26 03:18
上邪很棒啊,加油~~~~
其实小分队的速度练习不要求写回忆的,你算是超额完成任务
不过可以省些时间。你就读完自己在脑子里回想下大意就好,不用写出来。
作者: 199249712    时间: 2012-4-26 08:22
上邪很棒啊,加油~~~~
其实小分队的速度练习不要求写回忆的,你算是超额完成任务
不过可以省些时间。你就读完自己在脑子里回想下大意就好,不用写出来。
-- by 会员 babybearmm (2012/4/26 3:18:39)


啊~~~ 恩 下次不写了~~~
作者: 199249712    时间: 2012-4-26 09:43
9:43 一直在做小分队 挑挑拣拣的做了一些 上课去
15:18 一套GWD
16:41 总结改错完毕。。。T.T 猫了个咪。。为什么总错以前没错过的 还是没学扎实啊。。。
17:40.。看了会SC。。。看了会VB
21;16....终于总结完了今天的CR、。。。缺氧啊!!!!!!
21:55 总结了一篇阅读 整理了今天的生词 没来得及背。。。。。。。。。。。。今天又没完成任务。。。数学还没做吖啊 啊啊啊啊啊啊啊啊啊啊啊啊啊啊啊啊啊啊啊啊啊啊啊啊啊啊啊
大爷要来了 大爷都认识我了 大爷每次见我都说。。肿么又是你。。。
作者: 199249712    时间: 2012-4-26 16:53
Should Your Dog Be Watching TV? Well Pets | By DOUGLAS QUENQUA |  April 25, 2012, 12:20 pm 49 Comments DogTV

- - 转自nyt。。。其实我就是喜欢这只狗。。。。。。。。
Plenty of things will grab a dog’s attention: squirrels, tennis balls, funny smells, other dogs. But a TV channel?
Absolutely, say the makers of DogTV, the first cable network to deliver 24-hour programming for dogs. The idea, they say, is that flipping on the channel while you go out for the day will keep your pet stimulated, entertained and relaxed. Call it “Sesame Street” for those who will never learn their ABCs.
The shows on DogTV are actually three- to six-minute segments featuring grassy fields, bouncing balls and humans rubbing dog tummies. There are also segments featuring noiseless vacuum cleaners and muted doorbells to help make dogs more comfortable around such common household agitations.
Executives at the network say their programming is scientifically designed to appeal to dogs. “We have three years of research on how dogs react to different stimuli,” said Bonnie Vieira, a spokeswoman for DogTV.
For instance, she explained, “For dogs who suffer from separation anxiety, DogTV is a tool that might help ease them, so maybe they’re not getting into trouble, and they’re happier, more relaxed, when you get home.”
But can dogs actually watch, and benefit from, television? Like most questions regarding canine consciousness, the answer depends on whom you ask.
“I think a lot of this is to make us feel better as opposed to making the pet happier,” said Dr. Ann E. Hohenhaus, a staff veterinarian for the Animal Medical Center in Manhattan. “Your pet needs adequate exercise and an interesting environment. You cannot just put on the TV and hope your dog is going to get better.”
Still, if the dog is paying attention to the screen, odds are it likes what it sees. “If the dog wasn’t enjoying it, he would find something else to do, like nibble on the end of a sofa,” Dr. Hohenhaus said. In that way, dog-oriented shows “could be a component” in a program designed to alleviate separation anxiety.
In a test of DogTV at the Escondido Humane Society in California, the pets housed in a “behavior evaluation ward” — essentially a holding pen for new residents — found that exposure to the channel at least temporarily helped reduce barking and antsy behavior.
Whether your dog actually pays attention to the TV may have more to do with the screen than what’s on it, said Stanley Coren, a professor of psychology at the University of British Columbia. He should know of what he speaks: in 2007, he created a series of DVDs for canines called “The Dog Companion.”
“Dogs have terrific motion sensitivity,” Dr. Coren said, meaning that the optical illusion that makes still images on a TV appear fluid and won’t fool them as easily as it does humans. “For many dogs, that’s a turn-off. It doesn’t look real to them.”
To increase the chances your dog will pay attention, place the high-definition TV at the pet’s eye level, Dr. Coren advised. “Some people wrote to me and said, ‘This DVD didn’t work, my dog paid no attention to it,’” he said. “Well, a lot of people just plugged the image into their wall-mounted TV set, and the truth of the matter is, your dog is not going to look up there.”
But, like people, some dogs just aren’t that into TV, said Teoti Anderson, a former president of the Association of Pet Dog Trainers. “Two of my dogs do pay attention to the TV depending what’s on,” she said. “One of them couldn’t care less.”
If your dog does show interest, it probably can learn from what he sees on a television, Ms. Anderson said. Exposing a pet to muted versions of everyday irritants like vacuum cleaners and doorbells, for example, is a time-tested method for reducing the animal’s fear of them. But an important aspect of the technique is amping up the volume as the dog grows comfortable — so, depending how quickly a dog learns, the owner may want to hover nearby to turn up the DogTV volume.
But — of course — dog owners shouldn’t mistake TV time for quality time, animal behaviorists cautioned. “It definitely isn’t a substitute for play time with your dog,” Ms. Anderson said. “Exercise can solve a lot of behavioral problems.”
DogTV has been available through cable providers in San Diego since February and can also be accessed online. Its purveyors aim to put it on cable systems nationwide by the end of the year.
作者: babybearmm    时间: 2012-4-27 04:13
这篇文章好可耐啊,谢谢上邪分享~~~
想起google公司允许员工带狗狗去上班,太有爱啦~
作者: 199249712    时间: 2012-4-27 10:44
这篇文章好可耐啊,谢谢上邪分享~~~
想起google公司允许员工带狗狗去上班,太有爱啦~
-- by 会员 babybearmm (2012/4/27 4:13:04)



对啊对啊 太人性化了。。。再对比对比哈尔滨的禁狗令。。。前段时间VB上有好多好多哈奇士金毛求拯救 。。太悲哀了
作者: 199249712    时间: 2012-4-27 11:57
   Avatar 2: Made in China?  转自economics

     Apr 24th 2012, 7:29 by G.E. | BEIJING  
         
 

       
             
FOURTEEN years ago James Cameron’s film “Titanic” shattered box-office records in China, as it did nearly everywhere else in the world. Its impact was especially shocking in a market that was captive to a conservative, state-dominated film industry, with no ability to produce a blockbuster of its own. Mr Cameron’s ballyhooed “Avatar” broke China’s records again in 2009 and 2010, despite more than a decade of development. Now the film bureaucrats in Beijing have a chance to accomplish something that would have been unthinkable until very recently: co-producing Mr Cameron’s “Avatar” sequels.
Mr Cameron arrived in Beijing on Saturday and will soon be attending a screening of “Titanic 3D” at the Beijing International Film Festival (the re-release opened earlier this month to staggering sales in China). But his most important business will be conducted in private meetings, including with state-owned China Film Group. Speaking in an interview on Sunday, he said a priority of this trip was to explore a co-production deal with the Chinese firm on “Avatar 2” and “Avatar 3”. Mr Cameron says he would need to be satisfied in advance that his planned films would meet the approval of censors. If that key condition can be met, he is keen on the potential payoff. “There are economic advantages,” as he puts it.
The economic advantage he has in mind would be on the tail end, when the box office takings are divvied up. Mr Cameron does not need funding assistance for his films (a common reason for other foreigners in search of Chinese partners), but he would like China to share more of its blockbuster revenues with him. When “Avatar” made $200m in Chinese ticket sales, China was returning to Hollywood only 13% to 17% of the receipts on imported films, a far lower share than the American studios receive from other foreign markets. Going forward China will share up to 25% of the takings from imports, per an agreement announced during Xi Jinping’s visit to Los Angeles in February. That remains lower than Mr Cameron might be able to negotiate in a co-production deal. Chinese producers, after all, can collect up to 45% of the box office for domestic films, the 55% remainder going to satisfy the cinemas and distributors.
Mr Cameron’s meetings this week come shortly after the news that “Iron Man 3”, starring Robert Downey junior, will be a Chinese co-production. The gravitational pull of the Chinese movie market, nonexistent less than a generation ago, is now an undeniable force, sucking in all Hollywood blockbusters (and lesser projects) that venture within its event horizon. Hollywood studios, independent producers and directors regularly cycle through Beijing in search of partnerships with Chinese production houses—often seeking money to finance their movies, as well as access to a suddenly lucrative market.
This year China will surpass Japan as the world’s second-largest movie market, after America. Chinese box-office takings totalled 13 billion yuan ($2.06 billion) in 2011, an increase of 30% from 2010, which in turn had been more than 60% higher than in 2009. The number of movie screens has doubled in five years to more than 10,000 (and is projected to reach 15,000 in speedy fashion), and the new screens are mostly digital and 3D-capable. Meanwhile America’s market is stagnating. Takings in North America (America and Canada combined) declined by 4% in 2011, to $10.2 billion. Mr Cameron suggests that by the time “Avatar 3” is released later this decade, China may well rival America as the top movie market. That may be a stretch, but then just wait till “Avatar 4”; Mr Cameron calls it a possibility. He says he has stopped producing non-Avatar films or even considering non-Avatar scripts. “I’m in the Avatar business. Period, that’s it. I’m making ‘Avatar 2’, ‘Avatar 3’, maybe ‘Avatar 4’,” he says. “I think that within the Avatar landscape, I can say everything I need to say that I think needs to be said, in terms of the state of the world and what we should be doing about it.”
What Mr Cameron had to say in “Avatar”—about environmental exploitation, about the rights of people to their land—was rather political (Mr Cameron proudly declares it “not a subtle film”). The film resonated with some viewers in China as mildly subversive, and it did not receive quite the same blessing from Beijing as did “Titanic” (Jiang Zemin, then China’s top leader, was a fan). But it did not run afoul of censors. A famous scene in “Titanic”, in which Kate Winslet’s character poses nude for a drawing, was censored for the 3D re-release in China. Mr Cameron counts that as progress; he says that “somewhat” more was censored the first time the film was released in China. He surely has the leverage, with the value of “Avatar” as a franchise, to get the script assurances he would need to make a co-production work. He also says that he will not let any political concerns about China or its human-rights record interfere with his doing business here. “I’m going to do what’s necessary to continue having this be an important market for my films,” he says. “I’m going to play by the rules that are internal to this market. Because you have to.”
Indeed, as in many other industries, China has the market leverage to get what it wants from the foreign potentates who once dominated the film business. But what would China get in exchange for giving up some of its take at the box office to Mr Cameron? For one, Han Sanping, the powerful chairman of China Film Group, would affix his name to what could be one of the biggest blockbusters of all time, “Avatar 2” (and “3”, etc). Co-producing a James Cameron film would mark quite a symbolic turnaround for China, from the days of “Titanic”. Mr Han is often referred to in film circles as the godfather of Chinese film. If Mr Han wants a producer credit, Mr Cameron may find himself not terribly inclined to refuse.


噢- - 我还纳闷为什么卡梅隆回来北京电影节。。。原来如此
作者: 199249712    时间: 2012-4-27 16:25
老规矩早上在做小分队。。。还有自己摘抄的
14:21左右 做了一篇阅读总结
16:22做完一篇GWD。。
17:36断断续续改完错题 连着逛帖子聊天。。啊我太爱水了真是的。。
去吃饭。。好饿……饿……饿……~~~~(>_<)~~~~

20:05 读半小时的last题干--- 结果读到了现在
20:55总结两篇阅读。。。 我打算看会VB。。
21:36 做了20道PP07的题 错了一个以前没错的。。。真讨厌

晚上计划:
总结今天单词
作者: 199249712    时间: 2012-4-28 09:50
五一长假前的最后一天还要上课 这是一种神马命。。。 苦啊。。。
早上做了阅读小分队 读的比较快 没记内容 就过了个眼熟。。。
上楼咯 上课去。。。

14:27 被老师放鸽子 形势与政策的老师居然居然居然!!没有来上课。。。 于是我们全院5个班都被忽悠了。。。总结完了两篇阅读 并总结了5道CR
17:16 一套GWD。。。 并改错完毕 总结一下这一套的阅读
作者: 199249712    时间: 2012-4-28 10:08
Challenge for U.S. After Escape by China Activist     Supporters of Chen Guangcheng, via Associated Press
Chen Guangcheng, shown in an undated photograph, has been isolated since September 2010.


   BEIJING — The dramatic nighttime escape of a blind rights lawyer from extralegal house arrest in his village dealt a major embarrassment to the Chinese government and left the United States, which may be sheltering him, with a new diplomatic quandary as it seeks to improve its fraught relationship with Beijing.

The lawyer, Chen Guangcheng, one of the best-known and most politically savvy Chinese dissidents, evaded security forces surrounding his home this week and, aided by an underground network of human rights activists, secretly made his way about 300 miles to Beijing, where he is believed to have found refuge in the American Embassy, according to advocates and Chinese officials.        
An official in the Chinese Ministry of State Security on Friday said that Mr. Chen had reached the American Embassy, but American officials would not confirm reports that Mr. Chen had found shelter there.        
Mr. Chen’s escape represents a significant public relations challenge for the Chinese government, which has sought to relegate him to obscurity, confining him to his home in the remote village of Dongshigu and surrounding him with plainclothes security guards, even though there are no outstanding legal charges against him.        
The case also poses a major new diplomatic test for the United States. In February, the Obama administration was thrust into an internal Chinese political dispute when Wang Lijun, the former top police official from the region of Chongqing, sought refuge in the American Consulate in Chengdu. Mr. Wang revealed details about the killing of a British businessman, setting off a cascade of events that led to the downfall of Bo Xilai, who was the party chief in Chongqing and a member of China’s Politburo. American diplomats said they had determined that Mr. Wang’s case did not involve national security, and he was turned over to Chinese officials, prompting criticism from some in Washington about their handling of the case. Both sides insist Mr. Wang left of his own accord.        
But with Mr. Chen now believed to be on the grounds of the American Embassy in Beijing, administration officials are likely to be far more cautious in handling his case. His advocacy for the handicapped and for families subject to forced abortions and other coercive population control methods is widely known in the West. He also became a symbol of the deficiencies of China’s legal system after he was convicted of criminal charges in 2006 in a prosecution that Chinese lawyers — and even some officials in Beijing — felt made a mockery of China’s claims to be developing better legal norms.        
Mr. Chen, according to those who have spoken to him, slipped away on Sunday evening from his home in Shandong Province, where he has been held incommunicado since his release from prison in September 2010. Ai Weiwei, the artist and government critic who has also been subjected to residential detention, though far less draconian, said he had spoken to a friend who met with Mr. Chen in Beijing on Wednesday. The friend said Mr. Chen had climbed over a wall at night and evaded multiple lines of guards.        
“You know he’s blind, so the night to him is nothing,” Mr. Ai said the friend told him. “I think that’s a perfect metaphor.”        
Among those who helped Mr. Chen was He Peirong, a family friend who said Mr. Chen had planned his escape far in advance, staying in bed for long periods of time to trick guards into thinking he was too sick to walk. In an account she wrote on her microblog early Friday, Ms. He said that Mr. Chen had called her after fleeing the village. She said she then picked him up in her car, and they drove to Beijing. By late morning on Friday, Ms. He had been taken by public security agents from her home in Nanjing, according to Bob Fu, president of China Aid, a Christian rights group in Texas. Her microblog account was later deleted.        
A spokesman for China’s foreign minister on Friday said he had no information about the episode, but one intelligence officer expressed bewilderment that Mr. Chen had evaded his local government captors and had probably entered the embassy.        
“It’s still not clear how this happened,” the intelligence officer said. “Was this happenstance, or was it planned this way? Are there others planning to do the same?”        
The timing is especially inopportune for Beijing, given that it is preparing to welcome Secretary of State Hillary Rodham Clinton, Treasury Secretary Timothy F. Geithner and other American officials next week for the annual Strategic and Economic Dialogue.        
It also creates headaches for Washington, which has been eager to improve relations with the Chinese on various economic and security issues. Those efforts have lately paid dividends, with Beijing increasingly cooperating with American diplomatic moves to pressure Iran and North Korea over their nuclear programs. China has also shown a willingness to support United Nations efforts to broker a cease-fire in Syria.        
Mrs. Clinton has addressed Mr. Chen’s case on several occasions, most recently in a speech on Asian policy in November that prompted a sharp rebuke from Beijing. “We are alarmed by recent incidents in Tibet of young people lighting themselves on fire in desperate acts of protest,” she said then, “as well as the continued house arrest of the Chinese lawyer Chen Guangcheng. We continue to call on China to embrace a different path.”

On Friday, however, the State Department’s spokeswoman, Victoria Nuland, said she would make no comment about Mr. Chen’s escape or his whereabouts. The White House also declined to comment, and a scheduled briefing on Mrs. Clinton’s planned visit was postponed.

“Chen Guangcheng is a very strong candidate for asylum,” said Susan L. Shirk, a former State Department official who is now a professor at the University of California, San Diego. “A blind lawyer who is being persecuted for exposing forced abortions? I don’t think there’s any question about it.”        
But, as in the exploding scandal surrounding Bo Xilai, the Obama administration has sought to keep itself out of China’s internal politics.        
Rights advocates said Mr. Chen was not seeking to leave China, but would try to negotiate his freedom with the Chinese authorities.        
“He is reluctant to go overseas and wants only to live like a normal Chinese citizen,” said Mr. Fu, whose group had been in touch with Mr. Chen as recently as Friday morning.        
Shortly after news of Mr. Chen’s daring escape began circulating, a video appeared on YouTube on Friday, filmed in the days since he gained his freedom, in which he described life under house arrest. The video, in the form of an appeal to Prime Minister Wen Jiabao, detailed the abuse that he and his family suffered during their confinement and demanded that those responsible be brought to justice.        
He told of how his daughter was followed to school by three guards each day and how guards had kicked his wife for hours on end. “Prime Minister Wen, you owe the people an explanation,” he said. “Are these atrocities the result of local officials violating the law or a result of orders from the top leadership?”        
It is not the first time that Mr. Chen has sought to publicize the details of his confinement. Last year, he and his wife were reportedly severely beaten after a video they secretly recorded inside their home was smuggled out of the village and posted on the Internet. Friends say the subsequent abuse by their captors had left Mr. Chen in frail health.        
Mr. Chen, 40, is a self-taught lawyer, who was once lauded by the state media for his work defending farmers and the disabled. But he angered local officials after taking on the case of thousands of women who had been forcibly sterilized in Linyi County. During a brief trial in 2006, he was sentenced to 51 months in jail on charges of destroying property and assembling a crowd to disrupt traffic — charges that advocates say were trumped up, given that he was under house arrest at the time.        
After his release, he was taken directly to his family’s stone farmhouse, which was turned into a makeshift prison. His wife, and for a time his young daughter, were also confined inside the house, which was ringed by surveillance cameras, floodlights and a rotating cordon of guards equipped with walkie-talkies.        
Reporters, diplomats and Chinese activists who tried to visit Mr. Chen were violently repelled by guards at the entrances to Dongshigu. In December, the actor Christian Bale prompted a flurry of media coverage after he and a CNN crew were attacked outside the village.        
Rights advocates on Friday expressed concern for the safety of Mr. Chen and for his wife, Yuan Weijing, who activists said was left behind. Still, Mr. Fu of China Aid said he was optimistic that Mr. Chen might be able to negotiate his freedom. “The fact that he’s escaped will really shake up Chinese security forces,” he said. “It tells them that they are not almighty God.”
作者: 199249712    时间: 2012-4-28 17:21
18:17 总结完GWD的阅读。。也不知道老师能不能点名啊 今天五一前。。。

效率狂低。。。决定了  放弃逻辑大全 从逻辑链开始分类

晚上总结了两篇阅读 还有几道逻辑。。。忘了到第几题了。。T,T 我想看快乐大本营。。看完再总结。。。、

写一下晚上的任务:
20道SC不能丢  
20道CR 按照逻辑小分队的方法练习  并总结

好开心啊室友都回家了。。。终于可以在寝室看书了T.T
总结今天的单词
读LAST的题干半小时
总结RC。。。我为什么总错细节题? 奇怪。。。
作者: 199249712    时间: 2012-4-29 16:05
从下午开始看书的今天 早上一直在看快乐大本营。。。。
16:05 一套GWD
作者: 上邪    时间: 2012-4-29 23:01
改错完毕 总结RC CR 断断续续的今天 用的打电脑 哎,
睡觉。- -
作者: yakev6    时间: 2012-4-30 22:14
哼。
作者: 199249712    时间: 2012-5-1 13:07
哼什么吖。。。 大叔你在撒娇吗。。。
作者: 199249712    时间: 2012-5-1 13:09
罪恶了早上睡懒觉了。。
早上总结了3篇阅读
之后做了两个小分队 现在是13:08
苍了个天baby姐姐那个越障读的我泪都下来了。。

14:49
忍不住又做了一组大全。。。
=。=
16:07
又一组大全。。
17:15
在逻辑区瞎逛。逛到现在
18:00
总结了两篇RC。。。一切正确率都是我的幻觉。。。 哎。。。 不能大意啊。。。
作者: babybearmm    时间: 2012-5-1 14:46
上邪好赞啊~~~
嗯,我warning在前这篇越障很难 我自己那天计时读过后也有很多地方没看懂,不过论文章大意的话,我觉得你听了Sean Carrol那个演讲你就明白了。

罪恶了早上睡懒觉了。。
早上总结了3篇阅读
之后做了两个小分队 现在是13:08
苍了个天baby姐姐那个越障读的我泪都下来了。。
-- by 会员 199249712 (2012/5/1 13:09:00)


作者: 199249712    时间: 2012-5-1 14:49
上邪好赞啊~~~
嗯,我warning在前这篇越障很难 我自己那天计时读过后也有很多地方没看懂,不过论文章大意的话,我觉得你听了Sean Carrol那个演讲你就明白了。

罪恶了早上睡懒觉了。。
早上总结了3篇阅读
之后做了两个小分队 现在是13:08
苍了个天baby姐姐那个越障读的我泪都下来了。。
-- by 会员 199249712 (2012/5/1 13:09:00)


-- by 会员 babybearmm (2012/5/1 14:46:56)



好的好的 谢谢姐姐我晚上就看。。那阵读的我郁闷的 egg。。。brain。。。。
作者: teddybearj4    时间: 2012-5-1 17:05
上邪我才发现!!!!你的日志肿么写在这里哇!
作者: 199249712    时间: 2012-5-1 17:17
上邪我才发现!!!!你的日志肿么写在这里哇!
-- by 会员 teddybearj4 (2012/5/1 17:05:16)


抱抱teddy 好有缘分被发现啦。。因为偶懒得再开帖了。。。。。。。 嘻嘻
作者: 上邪    时间: 2012-5-1 20:27
20:26 连总结带瞎逛的到了现在 终于分类完毕OGCR RC还剩不几篇 都是前面的 不太想弄了。。
不总结了! 砸桌子ing。。 做GWD练PACE去
3+1+0 很开心诶阅读全对=0=

仔细改了下错题 3道SC都是上次没错的。。。果然做题的时候不仅要做题技巧还要集中精力啊
最后计划。。看一篇NYT 然后看姐姐给的连接 最后背背单词睡了。。。 睡懒觉果然好 晚上好清醒
琳姐今天特逗。。说手机让一黑摸去了。。然后跑到了小巷子里 她还不敢去追、 扯着扯着就扯到了南加州的枪案上0 0 RIP。。。。 想起来偶麻麻当时看到新闻后各种纠结 冲我嚷嚷一天你就别出去了。。。 麻麻今天生日。。麻麻生日快乐 劳动节生日 啧啧。。。一看就是苦命人 生我这么个糟心的闺女,,,
作者: 上邪    时间: 2012-5-1 23:29
16: GWD-29-Q8 模考错误
Which of the following, if true, provides evidencethat most logically completes the argument below?

According to a widely held economichypothesis,imposingstrict environmental regulations reduces economic growth.  This hypothesis is undermined by the factthat the states with the strictest environmental regulations also have thehighest economic growth.  This fact doesnot show that environmental regulations promote growth, however, since ______.

A.     those states withthe strictest environmental regulations invest the most in education and jobtraining

B.     even those states that have only moderately strictenvironmental regulations have higher growth than those with the least-strictregulations

C.     many states that are experiencing reduced economic growthare consideringweakening their environmental regulations

D.     after introducing stricter environmental regulations, manystates experienced increased economic growth

E.      even those states with very weak environmentalregulations have experienced atleast some growth



一道常识题- -

http://forum.chasedream.com/GMAT_CR/thread-592308-1-1.html?postid=15360168&searchmode=TopicContent&searchtext=imposing%20strict%20enviro

Common sense: Investment in Education and Job Training promotes growth in Economy.

C only says the those government are CONSIDERING deregulation. It does not prove that deregulation PROMOTES or WILL PROMOTE growth.

There is another facet of the problem one needs to consider: the main conclusion.

The author agrees with the widely held hypothesis and he is trying to refute an undermining evidence (most strict regulation is associated with the highest growth.) His conclusion is that "This fact does not show that environmental regulations promote growth," and your job is to find a premise which supports this main conclusion.

If you choose C, you are against the author's conclusion.                                    

作者: 上邪    时间: 2012-5-2 00:04
The monkey and the mouse 转自economics
     China’s film market is proving tough for foreign studios to crack              Apr 28th 2012              | SHANGHAI                    | from the print edition          
           
 

           
       Made in China?      
FOR years China’s home-grown hero, Monkey King, has had to compete on screen with the growing influence of an American mouse named Mickey. As Walt Disney and other foreign studios seek to enter China’s film market, and a new Disneyland theme park rises in Shanghai, the mouse’s position might look strong. But the monkey still holds plenty of cards.
China is now the world’s second-biggest film market after America. It has a booming home-grown film industry, making historical dramas and romantic comedies, but foreign blockbusters are the big money-earners. Although most films are pirated on release and viewed online and on dodgy DVDs, the rising middle classes are increasingly willing to fork out for a night at the cinema. Last year China’s box-office take rose by more than 30%, to over $2 billion, according to the Motion Picture Association of America. The number of cinema screens in China has doubled in five years, to nearly 11,000—again, second only to America. China’s box-office revenues may overtake America’s by 2020.
           In this sectionReprints
Related topics
   
 Yet China will not grant Hollywood the access it desires. Until recently only 20 foreign films could be screened at Chinese cinemas each year. In February the number increased to 34—though only if the extra 14 are shown in 3D or large format.
So, to guarantee their films are released in China, American studios are trying another ploy: seeking a Chinese partner. Co-productions are not classed as imports and so bypass the 34-a-year quota. They may have better luck being distributed, too.
A number of co-productions are under way. Walt Disney recently announced its first partnership with DMG Entertainment in Beijing to produce “Iron Man 3”, starring Robert Downey junior. This week, during a visit to the Beijing International Film Festival, James Cameron, a director, said he is looking for co-production opportunities in China for sequels to his film “Avatar”. The film remains the highest-grossing of all time in China. Chinese audiences also love Mr Cameron’s “Titanic”: the newly released 3D version took $105m in its first two weeks of release, double the takings in North America over the same period. But Hollywood’s dealings with China have been marred by allegations of corruption. On April 24th Reuters reported that America’s Securities and Exchange Commission had launched an investigation into whether American studios made illegal payments to Chinese officials.
China is difficult in other ways too. From the language barrier to hiring the cast (a percentage of whom must be Chinese), studios must adapt and some try to avoid offending China, even when their films are not co-productions. The invading army in a remake of “Red Dawn”, an American film, was changed from Chinese to North Korean in post-production.
But making a film that Chinese and Western audiences both want to watch is tricky. The 2010 remake of “Karate Kid” swapped Japanese karate for kung fu, yet the film performed badly in China, where audiences did not warm to a bully-boy depiction of the Chinese. And what will the need for Chinese-government approval do to the quality of co-productions made for the American market?
Dan Mintz, head of DMG Entertainment in Beijing and co-producer of “Iron Man 3”, describes China as both Hollywood’s saviour and its worst nightmare. Co-productions will not necessarily be filmed in China. The key is that Chinese producers will provide funding. But co-produced films are scrutinised by censors at both script and post-production stages. Films must comply or risk being bounced from cinemas, which would be business suicide in a market where nine-tenths of revenue comes from box-office receipts (in America, it is closer to 30%).
Government rhetoric suggests that the fledgling domestic market needs protection from American imports, at least long enough for it to grow in strength. Shanghai Film Group’s 3D remake of “Havoc in Heaven”, starring one Monkey King, was released in January and made just $8m at the box office. DreamWorks Animation’s “Kung Fu Panda 2” made more than $90m. The mouse may be commercially mighty, but the monkey has bureaucrats. And, for the time being, they are stronger.
作者: 上邪    时间: 2012-5-2 14:09
14:09 做完阅读小分队了 总结了生词 在逻辑区瞎逛了一会
15:01 alan哥把pp08图片破解给偶咯 撒花~~~~~ 上班的人就是不一样
15:58 总结了两篇阅读。。。现在这叫一个困啊。。。。。。。。。。。。。。。。。。。。。。。。。。。。。。。。。。。。。。。。。。。。。。。。
好困啊好困啊好困啊这样怎么行啊。。我报的可是下午场啊。。
深吸一口气 少女 冲向GWD吧争取突破昨天记录
18:08 3+1+3 sc已经死了= =|||

总结完了今天的RC。。。- - 艾玛 真的好绕啊。。。。
20:19 终于总结完了今天所有的题目。。。。。。(%……&%&……¥(&……*……*) 我痛恨总结!
休息一下一会去总结CR
00:34 看了一篇NYT 短的。。单词总结完毕。。好多啊啊啊啊啊啊啊
作者: teddybearj4    时间: 2012-5-2 18:22
我又欠小分队了……
上邪你的帖子里是做的你自己阅读的小摘录吗?
作者: 上邪    时间: 2012-5-2 19:21
恩、、、不过偶摘的比较没营养猫猫狗狗电影啥的。。本来想假装正经一点关注关注世界动态看了两篇就看不下去了。。。
作者: 上邪    时间: 2012-5-2 23:05
Harvard and M.I.T. Team Up to Offer Free Online Courses
By TAMAR LEWIN
Published: May 2, 2012
Facebook
Twitter
Google+
Email
Share
Print
Reprints

In what is shaping up as an academic Battle of the Titans — one that offers vast new learning opportunities for students around the world — Harvard University and the Massachusetts Institute of Technology on Wednesday announced a new nonprofit partnership, known as edX, to offer free online courses from both universities.

News, data and conversation about education in New York.

Join us on Facebook »
Follow us on Twitter »
Harvard’s involvement follows M.I.T.’s announcement in December that it was starting an open online learning project to be known as MITx. Its first course, Circuits and Electronics, began in March, enrolling about 120,000 students, some 10,000 of whom made it through the recent midterm exam. Those who complete the course will get a certificate of mastery and a grade, but no official credit. Similarly, edX courses will offer a certificate but will carry no credit.

But Harvard and M.I.T. are not the only elite universities planning to offer a wide array of massively open online courses, or MOOCs, as they are known. This month, Stanford, Princeton, the University of Pennsylvania and the University of Michigan announced their partnership with a new for-profit company, Coursera, with $16 million in venture capital.

Meanwhile, Sebastian Thrun, the Stanford professor who made headlines last fall when 160,000 students signed up for his Artificial Intelligence course, has attracted more than 130,000 students to the six courses offered at his new company, Udacity.

The technology for online learning, with video lesson segments, embedded quizzes, immediate feedback and student-paced learning, is evolving so quickly that those in the new ventures say the offerings are still experimental.

“My guess is that what we end up doing five years from now will look very different from what we do now,” said Provost Alan M. Garber of Harvard, who will be in charge of the university’s involvement.

EdX, which is expected to offer its first five courses this fall, will be overseen by a not-for-profit organization in Cambridge, owned and governed equally by the two universities, each of which has committed $30 million to the project. The first president of edX will be Anant Agarwal, director of M.I.T.’s Computer Science and Artificial Intelligence Laboratory, who has led the development of the MITx platform. At Harvard, Dr. Garber will direct the effort, with Michael D. Smith, dean of the faculty of arts and sciences, working with faculty members to develop and deliver courses. Eventually, they said, other universities will join them in offering courses on the platform.

M.I.T. and Harvard officials emphasized that they would use the new online platform not just to build a global community of online learners, but also to research teaching methods and technologies. Online courses with thousands of students give researchers the ability to monitor students’ progress, they said, identifying what they click on and where they have trouble. Already, a researcher from the Harvard Graduate School of Education, using the M.I.T. Circuits course, found that students overwhelmingly preferred to read the handwritten notes of Professor Agarwal rather than the same notes presented on PowerPoint.

Education experts say that while the new online classes offer opportunities for students and researchers, they also pose some threat to low-ranked colleges.

“Projects like this can impact lives around the world, for the next billion students from China and India,” said George Siemens, a MOOC pioneer who teaches at Athabasca University, a publicly-supported online Canadian university. “But if I were president of a mid-tier university, I would be looking over my shoulder very nervously right now, because if a leading university offers a free Circuits course, it becomes a real question whether other universities need to develop a Circuits course.”

The edX project will include not only engineering courses, in which computer grading is relatively simple, but also humanities courses, in which essays might be graded through crowd-sourcing, or assessed with natural-language software. Coursera will also offer humanities courses in which grading will be done by peers. In some ways, the new partnerships reprise the failed online education ventures of a decade ago. Columbia University introduced Fathom, a 2001 for-profit venture that involved the University of Chicago, the University of Michigan and others. It lost money and folded in 2003. Yale, Princeton and Stanford collaborated on AllLearn, a nonprofit effort that collapsed in 2006.

Many education experts are more hopeful about the new enterprises.

“Online education is here to stay, and it’s only going to get better,” said Lawrence S. Bacow, a past president of Tufts who is a member of the Harvard Corporation and a visiting professor at the Harvard Graduate School of Education. Dr. Bacow, co-author of a new report on online learning, said it remained unclear how traditional universities would integrate the new technologies.

“What faculty don’t want to do is just take something off the shelf that’s somebody else’s and teach it, any more than they would take a textbook, start on Page 1, and end with the last chapter,” he said. “What’s still missing is an online platform that gives faculty the capacity to customize the content of their own highly interactive courses.”
作者: 上邪    时间: 2012-5-2 23:10
Blind Chinese Dissident Leaves U.S. Embassy for Medical TreatmentBy JANE PERLEZPublished: May 2, 2012



BEIJING — Chen Guangcheng, the blind Chinese dissident who fled house arrest last month in a dramatic escape from security forces, left the American Embassy in Beijing on Wednesday after securing assurances from the Chinese government that he would remain safe, American officials said in the first account of his diplomatically tense six-day stay there.
Enlarge This Image



Pool photo by Shannon StapletonSecretary of State Hillary Rodham Clinton with the Chinese diplomat Dai Bingguo, left, in Beijing on Wednesday.
Related

Connect With Us on TwitterFollow @nytimesworld for international breaking news and headlines.
Twitter List: Reporters and Editors


Enlarge This Image


Jordan Pouille/Agence France-Presse — Getty ImagesChen Guangcheng was wheeled through a hospital in Beijing on Wednesday after leaving the American Embassy compound.
Readers’ Comments
Share your thoughts.


The officials described details of the negotiations between both governments and Mr. Chen as well as a telephone call to the dissident from Secretary of State Hillary Rodham Clinton after he left the embassy compound for treatment at a medical facility here.
Mrs. Clinton said in a statement that she was “pleased that we were able to facilitate Chen Guangcheng’s stay and departure from the U.S. Embassy in a way that reflected his choices and our values. I was glad to have the chance to speak with him today and to congratulate him on being reunited with his wife and children.”
“Mr. Chen has a number of understandings with the Chinese government about his future, including the opportunity to pursue higher education in a safe environment,” she added. “Making these commitments a reality is the next crucial task.”
Mr. Chen entered the American Embassy six days ago with the assistance of American officials because of the “exceptional circumstances, including his disabilities,” a senior American official told American reporters traveling with Mrs. Clinton. “On humanitarian grounds we assisted him and allowed him to remain on a temporary basis,” the official said.
Mr. Chen, a lawyer who had campaigned against forced abortions and sterilizations conducted as part of China’s policy of limiting families to one child, suffered an injury to his foot during his escape from his house in Shandong province last week and was walking with the help of a crutch, the official said.
During his time at the embassy, Mr. Chen adhered to his position that he was not seeking asylum in the United States but wanted to stay with his family in China as a free person, said the official, who was involved in the three-way negotiations that involved Mr. Chen and officials from the United States and China.
“He expressed his hope to stay in China and he never varied from that,” a second senior official involved in the negotiations, who briefed reporters, said.
On Wednesday afternoon, after Mrs. Clinton’s arrival about six hours earlier, and after the Chinese had made commitments to guarantee his safety, the American Ambassador, Gary Locke, asked Mr. Chen if he was ready to leave the embassy.
Mr. Chen, who speaks broken English, said in Chinese: “Let’s go,” one of the two American officials said.
As he left the embassy for the hospital, Mrs. Clinton phoned Mr. Chen in what the two American officials said was an emotional conversation since both Mrs. Clinton and Mr. Chen knew of each other but had never met.
At the end of the talk, according to one of the officials, Mr. Chen said to Mrs. Clinton: “I would like to kiss you.”
The officials said that during the negotiations inside the embassy, Mr. Chen at times would sit with the two main negotiators, holding each one of them by the hand. The two negotiators were the State Department’s legal adviser, Harold Koh, and the assistant secretary of State for East Asian and Pacific affairs, Kurt M. Campbell.
After driving a short distance to the Chaoyang Hospital from the embassy compound, Mr. Chen was reunited with his wife, Yuan Weijing, who was wearing a gray shirt decorated with a rainbow across the front, and their two children, whom he had not seen in some time, the officials said. Ms. Yuan had traveled from Shandong Province the previous day.
He was being treated by American and Chinese doctors, the officials said. Mr. Chen had agreed that his medical records be given to the Chinese doctors, they said.
Under the arrangement agreed to by the United States, China and Mr. Chen, he would be relocated to a different part of China from his hometown in Shandong, where he was under house arrest and where he says his family had been physically attacked, the officials said. The officials said he had been given a choice of seven locations agreed upon by the Chinese and Americans and that Mr. Chen had chosen Tianjin, an industrial port city east of Beijing.


(Page 2 of 2)


Mr. Chen would be allowed to enroll at a university to pursue his law studies, a profession in which he is self-taught, the senior official said. “He will have several university options,” one of the officials said.
Related

Connect With Us on TwitterFollow @nytimesworld for international breaking news and headlines.
Twitter List: Reporters and Editors



Readers’ Comments
Share your thoughts.


The American officials said they were satisfied with the pledges from the Chinese authorities that Mr. Chen, 40, would be allowed to live a normal life. The Chinese promised to report any actions against him, they said.
Precisely what the Chinese government offered as a way of protection for Mr. Chen was not immediately clear. The American officials went out of their way to praise the Chinese negotiators. They described them as working “intensely and with humanity.”
According to the American officials, negotiations began on April 26. The American negotiators met with their Chinese counterparts, led by the vice foreign minister, Cui Tiankai, at the Chinese Foreign Ministry, and relayed the issues to Mr. Chen at the United States Embassy. Mr. Chen never met directly with the Chinese officials, the American officials said.
There appeared to be no similar case in which a high-profile Chinese dissident had sought protection at the American Embassy and then returned to Chinese custody. American human rights officials and lawyers have often questioned whether the Chinese would provide the protection they promised in such a situation.
“This was not easy for the Chinese government,” one of the senior American officials said.
Only hours earlier, the crisis that has swirled around Mr. Chen seemed far from abating as China accused the United States of interfering in its affairs and demanded an apology from Washington for taking a Chinese citizen into the embassy “via abnormal means.”
“The Chinese side is strongly dissatisfied with the move,” the official Xinhua news agency quoted a Foreign Ministry spokesman, Liu Weimin, as saying. “The U.S. Embassy in Beijing has the obligation to observe relevant international laws and Chinese laws and it should not do anything irrelevant to its function.”
The two American officials declined to address the demand that the United States apologize for sheltering Mr. Chen and that the United States investigate the circumstance in which the embassy was used in what the Chinese said was an “abnormal” way.
“Our actions were lawful,” one of the American officials said.
Mrs. Clinton is in China for two days of scheduled talks with senior Chinese officials on economic and security matters.
She landed in Beijing shortly before 9 a.m. local time. Whether she took charge of negotiations was not immediately clear but Mr. Chen was admitted to the medical facility some hours after her arrival. Mr. Chen’s case will continue to overshadow the talks, known as the Strategic and Economic Dialogue, which are scheduled to begin Thursday.
But movement toward a resolution may ease some of the pressure. The Obama administration and the Chinese government have been anxious to ensure the case did not dominate the talks, which will cover subjects from North Korea to the global economy.
The last Chinese dissident to take refuge in an American diplomatic compound was Fang Lizhi, an astrophysicist, who walked into the embassy in Beijing with his wife in 1989, the day after the People’s Liberation Army crushed pro-democracy protests in Tiananmen Square.
The Chinese government regards foreign criticism of its human rights policies and practices as undue interference in its internal affairs, and it will almost certainly use the occasion of the talks to drive that point home, diplomats in Beijing said.
作者: yakev6    时间: 2012-5-3 00:25
来顶奇葩
作者: 上邪    时间: 2012-5-3 00:37
来顶奇葩
-- by 会员 yakev6 (2012/5/3 0:25:43)


奇葩攒了400多个单词还没背呢嘤嘤。。。。
作者: 上邪    时间: 2012-5-3 10:48
早上电脑坏了。。坏的很奇怪 网卡好的网线好的无线能上就是上不了本地连接 自己折腾了半天差点一个激动把驱动删了重装 忍住了打了客服 结果电话打到一半。。。电脑自己好了  太不给面子了。。。

10:48 小分队结束 今天的文章选的好棒啊><
15;55总结CR 接下来是模考时间。。。
17;41 SC1+2CR+ RC完全不确定了现在 争议题太多。。。
20:54 我。。。总结完了 16套的阅读简直太差了 争议好多啊 我从头到尾的翻论坛 好多解释还是不很满意。。
啊啊啊啊今天都干什么了啊效率太低了
单词终于背完了。。)&……(*%(*……(&) 吐血中
大晚上的读LAST真的很想死有木有。果断去睡觉!
作者: CCcarol    时间: 2012-5-3 14:36
so...如果今天我没有来逛逻辑区的话你是不是打算永远都不告诉我了!!!!!!!!!!!!!!!!!!!!!!!!!!!!!!!!!
作者: 上邪    时间: 2012-5-3 15:57
so...如果今天我没有来逛逻辑区的话你是不是打算永远都不告诉我了!!!!!!!!!!!!!!!!!!!!!!!!!!!!!!!!!
-- by 会员 CCcarol (2012/5/3 14:36:35)


偶错啦。 偶就是懒得新开帖就直接在这记啦。。
作者: 上邪    时间: 2012-5-4 10:29
04/29/2012            Are the Incentives of Colleges Aligned With Social Welfare? Becker                            The over 4,000 American private and public colleges and universities compete fiercely for students, faculty, and grants, and constitute the most competitive system of higher education in the world that provides both high quality and low quality programs. American universities are a magnet for postgraduate, and increasingly also for undergraduate, students from other countries. These two facts suggest that American universities (like Posner I use the word university to stand also for colleges) are doing a very good job of catering to the interests of students the world over. More generally, American universities are pretty successful in producing higher education that contributes effectively to social welfare, given the public policies that impinge on their behavior.
American public, private non-profit, and increasingly for-profit institutions of higher education compete hard for students and faculty. As a result, they offer a variety of courses, programs, and qualities of colleges and universities that range from a bare minimum program at many public community colleges to elite education at universities like Harvard, Stanford, and Chicago. These programs cater to students of varying qualities and with different interests. Students vote with their feet by choosing some institutions and programs over others, and by traveling long distances from other countries to attend American universities. This “voting” has made American universities responsive to the interests of students, which on the whole is a very good thing since these interests reflect changing job prospects and other changes in society.
In addition, American universities conduct much of the basic research conducted in the US, with support from the federal government and private gifts. That many young and older scientists and scholars from abroad compete to spend significant time at American universities is a good indication of the leading edge quality of this research. Perhaps they do not do “enough” basic research, but they do much more than universities elsewhere, and they would do even more if the federal government increased its support of university research.
American universities have been criticized because many of them engage in high-level competitive sports that involve heavy recruitment of student athletes. Since students and alumni like rooting for their school’s teams, these are perfectly appropriate activities for universities, aside from a couple of major problems. One is the exemption that the Supreme Court has granted to the obvious cartel-like behavior of the NCAA that uses its power to severely restrict compensation to student athletes, especially those in football and basketball. Universities should be forced to pay competitive prices for these athletes, not the much lower cartelized prices that the NCAA enforces. A much higher cost of star football and basketball players would induce some universities to tone down their emphasis on these sports, but many universities would still compete in these and other sports.
I agree with Posner that the federally financed student loan program needs significant modifications. More market-based interest rates on these loans are desirable, but in addition students under various circumstances should be allowed to borrow more than the current maximum limits on these loans. Especially students who attend expensive private universities may want to borrow more than they can at present, but most of them also receive high enough earnings later on to finance the interest repayment burden on these loans. It is no harder for most families to carry $100, 000 or more in student loans than it is for them to repay mortgage loans of comparable size.
However, students who are fettered with loans that they cannot repay should be able to discharge all or part of their loans through personal bankruptcy. To be sure, unlike mortgages, student loans do not have collateral that can be taken over by lenders in case of defaults on the loans. This is not so different than home ownership in the many states that do not allow the individuals declaring personal bankruptcy to be sued, although lenders can foreclose their homes. Despite the absence of collateral, workers who cannot repay their student loans should have the option of reducing the burden through discharging some of the loans through personal bankruptcy, the way other debt can be dischargeable through bankruptcy. To limit the abuse of this privilege, universities (including the for-profits) that make many student loans that end up being in arrears or discharged through bankruptcy should have their ability to make further loans severely constrained. This is already done to some extent, but tightening these constraints would force schools to be more careful in who they qualify for loans and the amounts they qualify for.
Having taught for almost all my adult life at American universities I am well aware of their many limitations. These include faculty who cater to students by easy grading and telling jokes, faculty who engage in vicious battles over trivial issues, faculty and administrators who are afraid to take stands against political correctness and the latest education fads, alumni and other donors who are cultivated for large gifts that really do not help a university’s mission, and so forth. Nevertheless, on the whole, American universities do an excellent job of providing up to date and diversified education for students of varying abilities and interests. Many of their “failures” are the result of bad incentives provided by federal and state support and regulation of university programs. Students the world over have voted for decades with their feet in favor of American universities against what is available in other countries.
作者: 上邪    时间: 2012-5-4 10:30
04/01/2012            Fracking and Self-Sufficiency in Gas and Oil-Becker                            More or less every American president starting with Dwight Eisenhower, and prioritized by Richard Nixon, called for American self-sufficiency in energy sources. In fact, America is now about self-sufficient in natural gas, and America’s oil imports have declined as a fraction of its total oil consumption from a peak of 60% in 2005 to about 50% in 2011. Part of the decline is due to the Great Recession’s effects on US output and automobile use. Another part is due to rising prices of oil that reduced oil imports, but increased spending on these imports. A third and growing part is due to increased domestic production of oil and especially gas that is likely to continue to grow rapidly during the next decade.  The main reason for the expansion in domestic gas and oil production is a technique called “hydraulic fracking”. Texas wildcatter George Mitchell was the most important person responsible for the development of the fracking method to extract gas from shale formations in the 1980s. This method uses large quantities of water under high pressure with added chemicals to crack open rocks and extract the gas, and sometimes oil, hidden in these rocks.   The cost of using fracking for natural gas extraction has become so competitive that most US natural gas production comes from fracking. As a result, the price of natural gas has fallen from a peak of about $10.80 per million BTUs to $2.20 currently. Instead of building terminals that could import liquefied natural gas, energy companies are now trying to export more natural gas. US natural gas inventories are so bloated there is a possibility that the price temporarily could be forced down toward $0, or even to a negative level.   Traditionally, a barrel of oil has sold for about 11 times the price of a million BTUs of natural gas. During the past few years, rising prices of oil and declining natural gas prices have raised that ratio to almost 50.No wonder there has been a rush to substitute gas for oil in electric power generation and in other activities.  Environmentalists have criticized the use of fracking techniques, and have pushed for bans or serious restrictions on their use. These critics claim that the techniques use too much water that could be used for other purposes, that it contaminates drinking water in areas surrounding fracking activities, and that it pollutes the air in surrounding neighborhoods. I am not in a position to authoritatively evaluate these environmental claims.  However, the chairman of Chesapeake Energy, a major fracking company, recently argued in a Wall Street Journal interview that fracking does not use so much water, and that in any case the company in most of its operations now recycles 90% to 100% of the water it does use. Furthermore, the EPA just dropped its claim that a different energy company contaminated drinking water in Texas. This is the third time in recent months that the EPA backtracked on claims that link fracking to pollution of surrounding water supplies. The size of the effects of fracking on air pollution is still an open question as the debate continues over this and other environmental effects of fracking.  Some political leaders have proposed a very bad idea:to restrict American output of oil and gas to use by American industry and consumers. If such laws were enacted and yet the US continued to import oil from abroad-which will be the case for the foreseeable future- restricting US production of oil to American use would have no effect on the domestic price of oil. The reason is that the global oil price would then determine domestic prices since no one would buy domestic oil if it were more expensive than imported oil, and no one would use imported oil if it carried a higher price than domestically produced oil.   On the other hand, suppose domestic production was so large that oil imports were unnecessary and exports of oil were prevented. Then domestic oil (along with natural gas) would sell below the world prices for these fuels. This would be bad for the domestic oil and gas industry because it would be forfeiting profits from selling some of its production abroad. Moreover, the relatively low domestic prices of these fuels would encourage greater domestic use that would lead to more pollution, although cleaner gas or oil would be substituted for dirtier coal in the production of electric power and in other uses.  Fracking has made the US self-sufficient in gas, and it is leading to reduced imports of oil. If this progress continues, before too long US consumption of oil as well as natural gas would not be drastically affected even by an entire breakdown of imports from the Middle East. The early progress in fracking techniques was very much aided by federal support and the work of engineers in the Energy Department. However, fracking was made into a profitable technique mainly through the ingenuity of people like George Mitchell in search of financial gains from finding ways to expand domestic production of gas and oil.
作者: 上邪    时间: 2012-5-4 10:54
10:54 复习了昨天的200个单词 做完阅读小分队 摘完了生词 看了两篇自己摘的文章 谢谢猴猴亲给的博客链接咩、、、
12:28 结束CR总结。??
13:32 忍不住又总结了一会。。。 今天把PP07-2总结完了。。不过一定要记得自己再看一遍哇,,不然白总结了
14:41总结PP074篇阅读。。 文章都很短

接下来进入模考时间。。 从数学开始万恶的数学 。。
数学结束 错8个。=。=  全部错在后面了啊啊啊啊啊啊 最后10个错了6个我还能不能行了、。老毛病啊啊啊老毛病啊 啊

16:39 开始GWD 果然模考的时候应该先看点别的把自己搞晕再开始才比较仿真
结束GWD15 3+1+5 阅读错毁了- - 开始改错。。。 = = 今天倒是挺好 错的跟上次一样。。
19:11终于改错改完了 。。去吃饭  我怎么套套GWD都错三个语法= = 好无语

晚上复习了早上的200个单词 联系了没多久的读题吧。看了看老管的无关方法 = = 感觉太笼统 很难操作 做了5道SC

未完成:继续做SC 并且总结
作者: 上邪    时间: 2012-5-4 21:14
作者态度题单词总结
2010-05-16 10:08
apprehensive忧虑的, 担心的, 恐惧的, 挂念的
annoyed生气的
analytical分析的, 解析的
amused愉快的, 开心的, 好玩的
alarmed恐吓的, 警告的
bitter苦的, 痛苦的, 怀恨的
conversational谈话的,供谈话的,口语的
concerned关心的
cautious谨慎的;非常小心的;细心的
contemptuous表示轻蔑的;傲慢的
critical批判的, 批评性的, 评论性的
confident确信的;有信心的;自信的
cynical愤世嫉俗的, 冷嘲热讽的; 玩世不恭的
defiant挑战的, 挑衅的, 目中无人的
detached超然的;公平的;公允的
distrustful不信任的, 怀疑的, 可疑的
disparaging蔑视的, 毁谤的, 轻视的
demanding过分要求的, 苛求的
disparaging轻蔑的, 贬低的; 毁谤的
disenchanted不再着迷的
defensive辩护的
enthusiastic热心的, 热情的
enlightened开通的,开明的
facetious幽默的, 滑稽的, 喜开玩笑的
frustrated失败的, 落空的
flippant无礼的;轻率的
genial愉快的; 亲切的, 和蔼的, 友好的
inquisitive好奇的;好问的
idealistic唯心论的, 空想主义的, 唯心主义者的
light-hearted轻松愉快的, 无忧无虑的
noncommittal不表示明确意见的, 不表态的;不承诺的;含糊的
neutral中立的,不偏向的
naive天真的
objective客观的
prejudiced有成见的;偏颇的
persuasive有说服力的
pragmatic实际的, 注重实效的
questioning质问的
resentful愤恨的; 愤慨的, 显然不满的; 易怒的
realistic现实主义的
scholarly精深的;博学的;学者风度的,学者派头的
speculative思索的; 推理的; 纯理论的;推测的;投机(性质)的; 冒风险的
skeptical怀疑的
sincere诚挚的, 真实的, 真诚的
supportive支持的,支援的
supportive支持的,支援的
tentative试验性的, 试探的, 尝试的, 暂定的
unsympathetic无同情心的,冷漠无情的
understanding谅解的,能谅解的
unpatriotic不爱国的, 无爱国心的
whimsical古怪的;异想天开的
作者: 上邪    时间: 2012-5-5 10:06
monkey expressions:
monkeyshines=tricks or foolish acts
monkey around=you are doing something when you do not know what you are doing or  have no firm idea of what to do
Monkey business=secret,illegal activities
monkey out of someone=acting foolish or silly.
had more fun than a barrel of monkeys= had a really good time
Monkey suits= clothes or uniforms soldiers wear
作者: 上邪    时间: 2012-5-5 10:13
10:13 阅读小分队结束 果然假期结束了大家都回来了 太吵了。。
14:05 做了不少语法 错误率还不错 发现还是有不少漏洞  
15:25背+复习了400个单词 狂点有道单词本点了一个半点 要吐血了。。刚点完 又提示我还有100个单词。。。能不能智能点。。。
16:23 总结3道阅读
17:57 做了一套数学 太逗乐了今天这正确率 29-37全错。。结果一共就错8个 = =。。。 还不如上次呢 至少前面能分担俩。。。 怒。。
21:10艾玛我今天太无趣了 一共做了50道语法。。。。  晚上10道RC+20道SC  

未完成:今天的语法改错还有不清楚的知识点没有查 GWD今天没来得及做就算了吧。。 晚上还要背单词 今天的小文章还没有摘。。= =||| 我的妈啊

哎呦艾玛、、= = 寝室人一回来就好吵。。 &*……))&( 姑娘们能晚上聊点有料的话题吗 唧唧杵唧唧杵的
作者: 上邪    时间: 2012-5-6 10:30
早上体能测试 来晚了
10:30 做了3个速度 不太想看了 直接上阅读题吧
13:05 做了好多阅读 没数 估计6篇吧 补完了今天的阅读小分队 看了会VB。。 开始SC之旅 首先从昨天的错题改起。。。我真是拖延症 。。哎
15;00终于总结完了语法点、。&(*%…………%&……*……*&)&(*&……%&(…… 又到了模考时间了。。。哎一天天的过的好快 进入数学。。。
16:05 30道DS结束。。错了5个 。。。。。。。。。。。。。。。。。。。。
19:12 无比憎恶文科院校的作业 形式化论文、 终于拼完了一篇 开始继续模拟生活。
GWD 偶来啦
20;35 状态各种不好 错2+1+4个 好烦
作者: 上邪    时间: 2012-5-7 17:23
17:22 今天一天课 早上做了小分队 中午开始背单词 背了500多个吧 到了现在 500多个也忘了大半了  老师无比扯淡 扯得够呛。。。快下课吧!!!!
19:14终于能静下来看会书了 30个语法 -错4 开始6篇阅读
20:22 做了4篇 不做啦 这次的速度倒是不错 1+1+0+1  3长一短 长的7分钟 短的5分钟 果然逻辑题不好啊。。阅读逻辑题老错
21:39 GWD13  1+1+0。。。 去改错 。。。 改完啦、、


未完成:CR没做。单词没背、SC错题没改 数学也没做啊啊啊啊啊啊!!
作者: yakev6    时间: 2012-5-8 00:31

作者: 上邪    时间: 2012-5-8 08:27
你到住的地方了??




欢迎光临 ChaseDream (https://forum.chasedream.com/) Powered by Discuz! X3.3